AGNP Midterm

¡Supera tus tareas y exámenes ahora con Quizwiz!

A 78-year-old woman comes to the office for a routine visit; she is accompanied by her daughter, who mentions that the patient has difficulty hearing conversations at the dinner table. The patient disagrees. However, on further questioning, a component of age-related hearing loss seems likely. On average, how long do patients have hearing loss before they receive medical attention? (A) 1-3 years (B) 3-5 years (C) 5-7 years (D) 7+ years

(D) 7+ years As life expectancy has increased, so has the proportion of the population living with age-related hearing loss. Its onset is insidious, and, typically, adults may not notice or disclose the condition for 7-10 years. Undiagnosed hearing loss can have dramatic consequences: affected individuals have a higher incidence of depressive symptoms, cognitive decline, falls, and need for assistance with activities of daily living. Only 10%-25% of adults with hearing loss receive treatment, in part because of underutilization of screening resources. Given the high incidence of hearing loss, patients ≥50 years old should be asked whether they perceive any difficulty with hearing. Screening measures can be as simple as a single question, such as, "Do you have difficulty with your hearing?" Other common measures with good sensitivity and specificity include the finger rub or whisper test, a multiple-item patient questionnaire (such as the Hearing Handicap Inventory Screening Questionnaire for Adults), or a handheld audiometer. They can be used to identify patients who may benefit from early referral for additional testing and treatment. Audiometry remains the gold standard for assessment of hearing loss. Several organizations have provided recommendations, although there is a general lack of consensus on who and when to test. The US Preventive Services Task Force currently finds insufficient evidence to screen asymptomatic adults. The American Academy of Family Physicians recommends questioning older adults about hearing loss and counseling them regarding the availability of treatment, when appropriate. The American Speech Language-Hearing Association recommends audiometric testing on request or if risk factors are present, and every 3 years after age 50. The Institute for Clinical Systems Improvement suggests that physicians should ask patients whether they suspect hearing loss and refer those who respond positively for formal audiometric testing. They further recommend screening patients who do not perceive any loss and to repeat screening every 2-10 years. The specific recommendations vary, yet it is important to recognize the substantial impact of hearing loss on aging adults and its potential for underdiagnosis.

A 79-year-old woman comes to the office to establish care. She has not seen a primary care provider in 6 years. She states that many years ago she had been told she had "high sugar" and "high pressure." On physical examination, blood pressure is 175/90 mmHg. Hemoglobin A1c is 12.3%. The patient is referred to ophthalmology for an eye examination. Retinopathy is diagnosed. Which one of the following is the appropriate interval for follow-up eye examination for this patient? (A) Annually (B) In 6 months, then annually (C) Every 2-4 months (D) Depends on the degree of retinopathy

(D) Depends on the degree of retinopathy

A 67-year-old woman comes to the office to establish care. She talks about her concerns regarding development of dementia: Her father had Alzheimer disease that developed when he was 88 years old. History includes hypertension, which she has had for 10 years and is well controlled by lisinopril 20 mg/d. She does not have diabetes. She is up-to-date on age-appropriate health maintenance. She walks 2-3 miles at least 5 times each week. She does not smoke and drinks wine 2-3 times in a week. She asks whether micronutrient supplementation will help prevent dementia. Which of the following should be recommended? (A) Antioxidant vitamins (eg, vitamin E, selenium) (B) Curcumin (C)Ginkgo biloba (D) No additional therapy

(D) No additional therapy Studies on the role of supplements—vitamins, herbs, nutraceuticals—in preventing cognitive decline have not shown consistent results. Over the years, interest in many of these supplements has come and gone. To date, none has been shown to prevent dementia or cognitive decline. (Option D) The PREADViSE (Prevention of Alzheimer's Disease by Vitamin E and Selenium) trial, in which 3,786 adults ≥60 years old were randomly assigned to vitamin E, selenium, vitamin E plus selenium, or placebo, showed no difference in the incidence of dementia (4.43%) among the four study arms. Curcumin, an active compound derived from turmeric, has known anti-inflammatory properties, but to date there is no strong study linking it to prevention of dementia or to slowed progression of cognitive decline. (Options A, B) In a 6-year randomized controlled trial across five academic centers in the US, including nearly 3,000 patients with normal or mildly impaired cognition, there was no reduction in incident dementia from Ginkgo biloba supplementation. Based on a Cochrane Review of 36 trials, the evidence is inconsistent and unreliable regarding predictable and clinically significant benefit of Ginkgo biloba for patients with dementia or cognitive impairment. (Option C)

The husband of an 82-year-old woman calls because his wife's behavior has changed over the last few days. She is confused and becomes agitated when he assists with ADLs. She will not eat because she thinks she is being poisoned. History: hypertension, depression, osteoarthritis, probable Alzheimer disease (diagnosed 2 years ago), urinary incontinence MMSE score was 22 of 30 at last visit 2 months ago. Medications: acetaminophen 325 mg four times daily, donepezil 5 mg/d, extended-release memantine 14 mg/d, hydrochlorothiazide 25 mg/d, lisinopril 10 mg/d, tolterodine 2 mg twice daily, and citalopram 20 mg/d. Donepezil and memantine were begun 2 years ago. Tolterodine was increased 1 week ago. Citalopram was increased (from 10 mg) 2 months ago. Laboratory findings: Blood urea nitrogen 18 mg/dL; Serum creatinine 1.1 mg/dL • Sodium 138 mEq/L • Glucose 81 mg/dL (consistent with prior measurements) • Urinalysis 0-5 WBCs/hpf, negative for bacteria and leukocyte esterase Which one of the following is most appropriate at this time? A. Discontinue tolterodine. B. Increase extended-release memantine to 28 mg/d. C. Start lorazepam 0.5 mg twice daily. D. Start risperidone 0.25 mg/d.

A. Discontinue tolterodine.

Which one of the following is true regarding the risk of prescribing psychotropic medications to patients with dementia? A. First- and second-generation antipsychotics increase both morbidity and all-cause mortality. B. Second-generation antipsychotics do not increase morbidity and all-cause mortality. C. First-generation antipsychotics do not increase morbidity and all-cause mortality D. First- and second-generation antipsychotics increase morbidity but not all-cause mortality.

A. First- and second-generation antipsychotics increase both morbidity and all-cause mortality. B. Second-generation antipsychotics do not

An 86-year-old woman comes to the office to establish care. History includes rheumatoid arthritis-associated interstitial lung disease, glaucoma, and anxiety disorder. She uses a cane when she walks. Current medications are methotrexate, folic acid, and sertraline, and she is on 2 L/min home oxygen. She says that she feels well and is not having symptoms of anxiety. She attends church every Sunday. Her daughter cooks and cleans for her. On physical examination, blood pressure is normal. There are dry crackles in both lungs. There is ulnar deviation of both hands, with no warmth or erythema of the metacarpophalangeal joints. Gait with cane is steady. Her records give no indication of advance care planning. Which of the following would be the most appropriate way to introduce that subject? (A) Ask her what brings her joy. (B) Recommend a "do not resuscitate" order. (C) Ask her to fill out a medical power of attorney. (D) Do not discuss advance care planning because she is religiously observant.

(A) Ask her what brings her joy. This patient has interstitial lung disease, a chronic, life-limiting condition. She would benefit from advance care planning to ensure that her medical care helps her maintain good quality of life and achieve her goals. This visit is to establish care; it is an opportunity to get to know her. An effective approach is to ask what is most important to her in life, such as asking what brings her joy (SOE=C). (Option A) Once there is some understanding of what makes life meaningful to this patient, it would be useful to explore what medical treatments might help her meet her goals. For patients with serious, life-limiting illness who seek to avoid prolonged hospitalization and aggressive interventions, it may be appropriate to recommend a "do not resuscitate" order. However, this recommendation would be premature without first learning more about the patient's values and goals. (Option B) A medical power of attorney is a type of advance directive. It is a legal document that designates a health care agent to make decisions on a patient's behalf, and specifies whether it should be put into effect immediately or only in the event that the patient has an incapacitating medical condition. All patients should be encouraged to designate a health care agent, but at this appointment it is more important to gain a sense of what the patient values. (Option C) Spirituality is important to many patients, especially when discussing advance care planning. Many health care providers fear that religiously observant patients might be hesitant to discuss advance directives. However, in a recent qualitative study of religious leaders from a variety of backgrounds, leaders were supportive of discussions of advance directives, especially if spiritual considerations are included and acknowledged (SOE=C). (Option D) A number of tools have been developed to assist in advance care planning, one of which, "PREPARE For Your Care," has been validated for use with Spanish-speaking older adults (SOE=B). As of 2016, Medicare reimburses physicians for 30-minute discussions on advance care planning. The discussions may take place during a patient's annual wellness check or at a separate visit.

A nurse practitioner requests a geriatrics consultation because she would like to reduce the medication burden for her patient, an 80-year-old man with depression, osteoarthritis, and heart failure. He currently takes 12 prescribed medications and 3 supplements. He lives 90 minutes away from the closest consulting geriatrician's office, and he does not have internet access in his rural community. The nurse practitioner is his primary care provider. Which one of the following would be most appropriate in this situation? (A) Asynchronous geriatrics electronic consultation (e-consult) (B) In-person comprehensive geriatric assessment (C) Remote patient monitoring (D) Synchronous televideo consultation

(A) Asynchronous geriatrics electronic consultation (e-consult) In this case, the consulting physician is asked to provide recommendations to the nurse practitioner for reducing polypharmacy for her patient. This type of targeted question can be answered in an asynchronous encounter. Clinicians providing geriatric specialty care through e-consults may find that they can offer brief input more efficiently and widely through e-consults than through traditional consultations that involve face-to-face or televideo evaluation of the patient. Electronic consultation allows for rapid, direct, and documented communication of recommendations from the consultant to the health care provider. A Veterans Administration program for evaluation and treatment of osteoporosis after hip fracture showed that an e-consult service modestly improved the rate of osteoporosis treatment among patients with a recent fracture. The results suggest the feasibility of asynchronous e-consults for the purpose of medication intervention. Medicare will reimburse for e-consultations. (Option A) Questions that require complex assessment or discussion of risks, benefits, priorities, and goals of care may not be suited to an e-consult: the time spent may not be adequately compensated, and complex care plans may be difficult for the primary provider to implement. In-person comprehensive geriatric consultation would provide an opportunity to intervene and improve polypharmacy. However, the patient's primary care provider has requested a consultation to reduce polypharmacy, not necessarily to provide a comprehensive assessment. Further, comprehensive assessment would be difficult to arrange for this patient, given how far he lives from the consulting geriatrician. Comprehensive geriatrics consultation may be conducted by televideo, but the lack of internet access in the patient's community precludes this option. (Options B, D) Remote patient monitoring combines remote collection of health care data with a mechanism for patient-clinician communication; it has demonstrated efficacy in specific situations. For example, among older adults undergoing chemotherapy, patients who report between-visit symptoms electronically have greater health-related quality of life and fewer emergency room admissions than patients who do not have the opportunity to report symptoms electronically (SOE=A). However, there are few data related to remote patient monitoring for assessment or intervention in polypharmacy. (Option C)

A 68-year-old woman has difficulty understanding clients in her law practice. She has worn hearing aids since age 50, when severe to profound sensorineural hearing impairment developed in each ear. She recently underwent a complete hearing examination; hearing status is unchanged, and audiometric test results remain consistent with sensorineural hearing loss associated with long-standing cochlear damage. There is no history of tinnitus, dizziness, diabetes, cardiovascular disease, or cognitive decline. Which of the following is the best recommendation? (A) Bone-anchored hearing aid (B) Cochlear implant (C) Hearing assistive technology for use with clients in the office (D) Referral for aural rehabilitation to learn lip reading

(B) Cochlear implant This patient would benefit from a cochlear implant, a treatment for severe to profound deafness appropriate for patients who derive little subjective or objective benefit from hearing aids. Objective benefit is assessed by performance on speech recognition tasks. About 10% of older adults with hearing loss have impairment sufficient to qualify for an implant, yet most of these candidates are not identified or referred. Unlike hearing aids, which work by amplifying sound, cochlear implants bypass damaged areas of the inner ear and transmit signals directly to the brain via undamaged parts of the auditory nerve. The implants replace the function of the damaged hair cells in the cochlea by converting acoustic energy into electric energy, which is picked up by the fibers of the eighth cranial nerve. A cochlear implant consists of a surgically implanted electrode array and a receiver/stimulator. There is an external portion as well, which includes a microphone, speech processor, and transmitter coil. (Option B) Cochlear implants have become an accepted treatment for adults with age-related hearing loss or progression of early-onset hearing loss and are effective for most patients. Healthy older adults can undergo the surgical procedure with minimal risk. A 2010 study demonstrated that a bimodal approach (hearing aid for one ear, cochlear implant for the other ear) provides superior results on tests of speech understanding when compared with a cochlear implant alone (SOE=A). Patients who receive a cochlear implant should continue to wear a hearing aid in the other ear, because binaural hearing is important to speech understanding. The implant is placed in the more impaired ear; the better ear is fit with a hearing aid. If hearing loss progresses in the better ear, patients may benefit from an implant in that ear as well. In a study of 56 adults who had similar hearing test scores before cochlear implantation, adults ≥65 years old and adults between 18 and 64 years old showed improvement in hearing 1 year after implantation (SOE=A). Regardless of age at implantation, higher test scores before surgery predicted higher test scores after. However, older adults performed more poorly on some speech perception tests at follow-up 1 year later. These data suggest that adults with significant progressive age-related hearing loss might consider undergoing implantation at a younger age, thereby maximizing performance after implantation. A bone-anchored hearing aid is appropriate for older adults with single-sided deafness or chronic middle-ear disease that precludes use of hearing aids. The outcome of surgery in older adults is favorable, and the rate of implant loss is comparable to that of the overall population of individuals who undergo surgery for a bone-anchored hearing aid. There appears to be a low risk of severe skin reactions or skin thickening around the implant. (Option A) Many patients benefit from support groups that facilitate coping with chronic conditions. The Hearing Loss Association of America (http://www.hearingloss.org/) provides support, information about community resources, tips for coping with hearing loss, and communication strategies. For the patient in this case, hearing assistive technology may help her understand clients in an office setting but would not be adequate to compensate for her degree of hearing loss in other venues. Hearing assistive technology would remain an option for any uncompensated needs after the cochlear implant. (Option C) It is difficult for people who have communicated orally their entire life to begin to learn lip reading late in life. Even if this patient has learned some lip-reading skill because of her long-term hearing loss, it would be inadequate for her current professional needs. (Option D)

A 79-year-old woman comes for follow-up 1 week after hospitalization for an exacerbation of heart failure. History includes atrial fibrillation, hypertension, systolic heart failure (New York Heart Association class III), and coronary artery disease. Current medications are furosemide 20 mg twice daily, potassium 10 mEq/d, lisinopril 10 mg/d, warfarin 2 mg/d, metoprolol succinate 50 mg/d, and, since hospital discharge, spironolactone 12.5 mg/d. On examination, blood pressure is 122/74 mmHg and heart rate is 70 beats per minute. Weight is stable and she has returned to baseline function. Laboratory findings (on hospital discharge ? today): Sodium 137 mEq/L ? 139 mEq/L Potassium 4.7 mEq/L ? 5.3 mEq/L Blood urea nitrogen 20 mg/dL ? 21 mg/dL Creatinine 1.1 mg/dL ? 1.3 mg/dL Which of the following is the most important next step? (A) Discontinue spironolactone. (B) Discontinue potassium supplement. (C) Hold lisinopril for now. (D) Change furosemide schedule to 20 mg/d.

(B) Discontinue potassium supplement. The increase in serum potassium is likely due to the addition of spironolactone. When spironolactone is introduced, serum potassium levels increase on average by about 0.3 mEq/L. The potassium supplement should have been discontinued when spironolactone was started. This patient is at particular risk of hyperkalemia with spironolactone because, in addition to a potassium supplement, she takes an ACE inhibitor (lisinopril) and she has kidney impairment. Discontinuing the potassium supplement will likely reduce the serum potassium level, obviating the need to decrease the dosage of lisinopril. (Option B) Although ACE inhibitors can cause hyperkalemia, it is prudent to continue patients on this therapy because mortality is reduced (SOE=A). ACE inhibitors improve survival and reduce morbidity in patients with left ventricular ejection fraction <40% (SOE=A). Additionally, the patient's weight is stable, and her function has returned to baseline, so there is no indication for decreasing or holding the lisinopril dosage. Increasing the dosage of lisinopril to a target dose of 20-40 mg/day would be optimal in managing her heart failure but should only be considered once her potassium has returned to normal and her blood pressure can tolerate it. Hyperkalemia often limits the dose of lisinopril that can be used. (Option C) Spironolactone use is associated with reduced mortality and fewer exacerbations of heart failure when used in patients with New York Heart Association class III or IV disease who were recently hospitalized for heart failure. Thus, it is an appropriate agent for this patient (SOE=B). Serum creatinine levels can increase by up to 20% with the use of aldosterone antagonists; this increase is not a reason to discontinue the spironolactone. In general, if the estimated creatinine clearance is <30 mL/min, spironolactone is not advised. ((Option A) Although taking furosemide once daily may be appropriate, reducing the furosemide dosage will not reduce potassium levels, while stopping the supplemental potassium would. (Option D)

A 65-year-old woman comes to the office for follow up after hospitalization for a clinical vertebral fracture (L4). History includes stage 3B chronic kidney disease (CKD), hypertension, and diabetes. Current medication is lisinopril 10 mg/d; the diabetes is controlled by diet. She underwent menopause at age 54. She does not drink alcohol, and she stopped smoking tobacco several years ago. Laboratory findings include normal intact parathyroid hormone level; phosphate level is >3 mg/dL. Results of dual-energy x-ray absorptiometry (DXA) are T-score of -1.5 at the lumbar spine, -1.3 at total hip, and -2.0 at femoral neck. Calculated FRAX® (Fracture Risk Assessment) score indicates 18% risk of major osteoporotic fracture and 2.8% risk of hip fracture. Bone biopsy done during hospitalization showed low bone formation rate with rare osteoblasts and osteoclasts. There was no defect in mineralization or trabecular microarchitecture. In addition to ensuring adequate calcium and vitamin D intake, which of the following is the most appropriate treatment for her bone health? (A) No treatment is indicated. (B) Oral alendronate (C) Raloxifene (D) Subcutaneous teriparatide

(B) Oral alendronate This patient's bone mineral density (BMD) suggests osteopenia, yet the evidence of fragility fracture signifies osteoporosis that requires treatment (SOE=A). BMD and FRAX® score are useful tools but do not exclude osteoporosis. Further, this patient has CKD, and CKD and osteoporosis are co-prevalent: up to 10% of women with an estimated glomerular filtration rate (eGFR) <60 mL/min have osteoporosis, and 21.3% of women 70-90 years old with osteoporosis have an eGFR <35 mL/min. In studies, patients with kidney impairment were older, shorter, weighed less, had been postmenopausal longer, and had lower baseline lumbar spine and femoral neck BMD than patients with normal kidney function (SOE=B). Given the presence of CKD, the possibility of CKD-mineral and bone disorder (CKD-MBD) complicates the decision regarding treatment. This patient has no evidence of CKD-MBD, as established by her normal intact parathyroid hormone level, absence of hypophosphatemia, and bone biopsy results. Therefore, she is a candidate for pharmacologic treatment. (Option A) Many of the therapeutic agents used to treat osteoporosis are affected by kidney function. It is generally agreed that treatment of osteoporosis for patients with stage 1-3 CKD without evidence of CKD-MBD is the same as that for patients without CKD. However, treatment should be modified for patients with stage 4 or 5 CKD and for CKD patients on dialysis who have osteoporosis. This patient has stage 3B CKD without CKD-MBD and, thus, should be treated in the same way as the general population. Bisphosphonates (such as alendronate and zoledronic acid) are considered first-line therapy because of their established efficacy, cost, and long-term safety data (SOE=B). Raloxifene may play a role in treatment for women at increased risk of breast cancer but there is no indication of that risk for this patient. Teriparatide is recommended for women at high risk of fracture, such as those with severe or multiple vertebral fractures. Denosumab is not listed as an option but would be a potential alternative first-line agent. (Options B, C, D)

A 69-year-old man comes to the office to establish care. History includes prostate cancer and hypothyroidism; he is a former smoker (50 pack-years). Medications include leuprolide 7.5 mg IM monthly and levothyroxine 150 mcg orally daily. His wife is being treated for osteoporosis and she wants to know whether her husband should undergo a screening assessment. On physical examination, BMI is >30 kg/m2. Which of the following is the strongest risk factor for osteoporosis for this patient? (A) Androgen deprivation therapy (B) History of tobacco use (C) Thyroid replacement therapy (D) Obesity

(A) Androgen deprivation therapy Osteoporosis is a problem in older men, but there are limited data to inform screening guidelines. Literature review suggests that the most important risk factors for osteoporotic fractures in men are age ≥70 years old and low body weight (BMI <25 kg/m2 or weight <70 kg [154 lb]) (SOE=A). Other risk factors include weight loss, physical inactivity, corticosteroid use, previous osteoporotic fracture, and androgen deprivation therapy (SOE=A). Androgen deprivation therapy (pharmacologic or by orchiectomy) is a strong predictor of both osteoporosis and fracture. (Option A) Other risk factors for osteoporosis in men have been reported, including cigarette smoking, alcohol use, insufficient vitamin D and calcium intake, respiratory disease, and thyroid replacement therapy. However, the strength of association is inconclusive in most cases. The possible risk factors have plausible physiologic rationales, and some are based on data in women or inconsistent data in men. Obesity is not known to be a risk factor for osteoporosis. (Options B, C, D)

John Rowe and Robert Kahn first conceptualized successful aging over 30 years ago. It is one of the best known and applied old age-positive models. In addition to preservation of cognitive and physical function and maintaining engagement in life, which of the following were included in their model? (A) Avoiding disease and disability (B) Avoiding toxins (C) Avoiding stress (D) Practicing religious faith

(A) Avoiding disease and disability Rowe and Kahn's model of successful aging remains influential. They describe three components of successful aging: Low probability of disease and disability (Option A) High cognitive and physical function Engagement in life (interpersonal relationships and productive activity) The model was a turning point in the geriatrics field, which began to view older adults as able and valuable society members. It portrayed the impairments associated with aging as not irreversible and uncontrollable, as had been understood to that point. Previous assumptions held that, in the absence of disease and disability, other changes in physical function, such as mild memory impairment and increases in blood pressure and blood glucose, were normal. A criticism of the model is that it is static rather than dynamic and does not consider developmental processes and trajectories as well as intergenerational and spiritual aspects. Avoiding toxins, stress and continued religious practice are not part of the model. (Options B, C, D)

Use of dietary supplements among older adults continues to be high in the US, without proven benefit for most supplements. Which of the following conditions has the best evidence for antioxidant vitamin supplements slowing progression of disease? (A) Macular degeneration (B) Mild cognitive impairment (C) Sarcopenia (D) Cardiovascular disease

(A) Macular degeneration Oral supplementation with the Age-Related Eye Disease Study (AREDS) formulation of antioxidant vitamins slows progression of both wet and dry forms of age-related macular degeneration (SOE=A). The formulation includes vitamins C and E, β-carotene, and zinc. (Option A) The AREDS2 trial evaluated the utility of replacing β-carotene with the carotenoids lutein and zeaxanthin and of adding omega-3 fatty acids (DHA and EPA) to the AREDS vitamin formulation, but the trial did not show either further decrease in risk of age-related macular degeneration or reduction in progression of early nonexudative macular degeneration (SOE=A). By eliminating β-carotene, the AREDS2 preparation is deemed safe for smokers. In multiple large randomized clinical trials of older adults with normal cognition or mild cognitive impairment, supplementation with a variety of antioxidant vitamins has shown no impact on cognitive change or incident dementia over follow-up times that have ranged from 7-10 years (SOE=A). (Option B) A systematic review has shown that multinutrient supplements improved chair rise time and handgrip strength in older frail adults (SOE=B), but no conclusive studies have shown efficacy of vitamins or antioxidants in frailty. Sarcopenia is part of the frailty syndrome; because vitamin D plays a critical role in muscle health, supplementation has been studied in seven randomized controlled trials as a therapy for sarcopenia, with mixed results. (Option C) A 2017 meta-analysis suggests that vitamin C supplementation does not reduce major cardiovascular events (SOE=A). This finding is consistent with the lack of benefit seen with the use of vitamin C for primary as well as secondary prevention of cardiovascular events. (Option D) Routine use of vitamin E to prevent cancer, cardiovascular disease, dementia, or degenerative eye disease is unsupported by evidence. For some conditions, such as prostate cancer or cerebral hemorrhage, the risk may be increased. Vitamin E supplementation may also increase the propensity for bleeding in older adults who are taking aspirin or anticoagulants such as warfarin.

An 88-year-old woman woke up last week with distorted vision in her right eye that has not resolved. Last year, an ophthalmologist diagnosed a dry form of macular degeneration associated with large drusen. She was treated with the AREDS2 formulation of eye vitamins, but she stopped after 1 month because she did not consider it "real medicine." History includes long-term tobacco use, and she continues to smoke. Visual acuity is 20/800 in the right eye and 20/100 in the left eye. Which of the following is the most appropriate course of action? (A) Explain that the AREDS2 vitamins slow vision loss in macular degeneration. (B) Explain that the AREDS2 vitamins combined with smoking cessation may prevent blindness. (C) Refer her to an eye specialist with expertise in low vision. (D) Arrange for her to see an ophthalmologist this week.

(D) Arrange for her to see an ophthalmologist this week. Hard drusen—submacular yellow lipoprotein deposits—do not typically cause loss of vision but are a marker for increased risk of conversion to the wet form of age-related macular degeneration (AMD), which is characterized by angiogenesis or choroidal neovascularization. Development of choroidal neovascularization is signaled by sudden loss or distortion of vision, as this patient has experienced. Because untreated disease can lead to severe central vision loss, this patient requires urgent evaluation. (Option D) The second Age-Related Eye Disease Study (AREDS2) found that the risk of choroidal neovascularization could be reduced when patients with larger, more numerous drusen (high-risk drusen) take a combination of zinc, cupric acid, lutein, zeaxanthin, and vitamins C and E twice daily. Vascular endothelial growth factor (VEGF) inhibitors are used to treat neovascularization in wet AMD, preserving vision in most patients and restoring vision in a significant minority (SOE=A). VEGF inhibitors are administered intraocularly by ophthalmologists. AREDS2 is recommended for patients with intermediate or advanced dry AMD and at all stages of wet AMD. This patient would benefit from AREDS2 therapy, but the more urgent need is for immediate assessment by an ophthalmologist and consideration for vision sparing/restoring therapy with a VEGF inhibitor. Smoking increases the risk of macular degeneration. Smoking cessation combined with AREDS2 treatment is recommended; whether this will prevent blindness is not known, but unlikely. It is too early to make a low-vision referral for this patient, and she may regain sufficient vision with VEGF therapy to make that referral unnecessary. (Options A, B, C)

A 75-year-old man comes to the office for his annual wellness visit. He is physically active and lives independently in an apartment. He has no significant medical history and has never fallen. His vitamin D level 6 months ago was normal. His only medication is aspirin once daily. Which of the following is the most appropriate vitamin D supplementation for this patient to prevent future falls? (A) No vitamin D supplement needed. (B) Prescribe vitamin D3 800 IU/d. (C) Prescribe vitamin D2 50,000 IU/week. (D) Prescribe vitamin D3 400 IU/d with calcium 1,000 mg.

(A) No vitamin D supplement needed. The US Preventive Services Task Force recommends against vitamin D supplementation to prevent falls in community-dwelling adults ≥65 years old who do not have vitamin D deficiency or osteoporosis and who are not at high risk of falls or fractures (grade D recommendation). For community-dwelling adults >65 years old who are at high risk of falling, only exercise and multifactorial interventions (grades B and C, respectively) have been shown to reduce falls. Multifactorial interventions address risk factors such as postural hypotension, home hazards, polypharmacy, and vision abnormality.

An 80-year-old woman has recently had occasional episodes of feeling "off balance and wobbly." She fell once in the past month when she lost her balance while walking at a mall where she goes to exercise. There has been no recent respiratory, gastrointestinal, or urinary tract infection. History includes osteoarthritis of the hands, for which she occasionally takes acetaminophen. She takes no other medications. On examination, blood pressure is normal, with no orthostatic hypotension or tachyarrhythmia. There is no evidence of depression or cognitive decline, as assessed using the Patient Health Questionnaire-2 and Mini-Mental State Examination. Visual acuity is 20/30 bilaterally. The whisper test result is positive. Multifactorial fall risk assessment, including the Timed Up and Go test, is unremarkable. Which one of the following is the most likely cause of her symptoms? (A) Saccular and semicircular canal dysfunction (B) Labyrinthitis (C) Vestibular neuritis (D) Utricular dysfunction

(A) Saccular and semicircular canal Loss of vestibular function is prominent among older adults. Use of the head impulse test, a measure of semicircular canal function, identified abnormalities in 35% of adults >70 years old and in 66% of adults ≥80 years old (SOE=B). Findings on the Modified Romberg Test, a global measure of the ability to use vestibular information, suggest a prevalence of vestibular dysfunction ranging from 59% to 69% among adults 70-79 years old and from 85% to 100% among adults ≥80 years old (SOE=B). This patient's presentation and symptoms are typical of saccular and semicircular canal dysfunction. (Option A) The inner ear includes the cochlea, saccule, semicircular canal, and utricle. The cochlea provides the auditory function. The saccule is an otolith organ involved in detecting vertical linear movement and sensing gravitational change (eg, vertical acceleration when going up in an elevator). The semicircular canals detect rotational movement of the head, and the sensory cells in the utricle are sensitive to change in horizontal movement. In older adults with high-frequency hearing loss, abnormal amplitudes are evident on cervical vestibular-evoked myogenic potential (c-VEMP) testing in response to air-conducted sound, which assesses saccular function. The association between hearing loss and reduced saccular function holds even after adjusting for shared risk factors, such as increasing age and noise exposure (SOE=B). In contrast, hearing loss is not associated with utricular function, confirming preservation of utricular structures with aging. Possibly, the association between cochlear and saccular functions may be because of their shared embryologic origin. It has been speculated that presbycusis (age-related hearing loss) may be a cochleosaccular degenerative process in which saccular otoconia reach the cochlea, affecting the cochlear base that subserves high-frequency hearing. (Option D) Labyrinthitis is an inflammation or swelling of the labyrinth of the inner ear. It is triggered by upper respiratory infection, typically viral but in rare instances bacterial. Although labyrinthitis may be associated with temporary hearing loss and tinnitus, its main symptom is vertigo, or the sensation of spinning or whirling. In labyrinthitis, symptoms occur without warning about 2 weeks after an upper respiratory infection or influenza. It is occasionally accompanied by nausea. Symptoms tend to subside after 1 month. (Option B) Vestibular neuritis is an inflammation of the vestibulocochlear nerve, the nerve that sends information about balance and head position from the inner ear to the brain. Whereas labyrinthitis involves swelling of both branches (vestibular and cochlear) of the vestibulocochlear nerve, neuritis involves swelling of the vestibular branch only. Hence, symptoms of neuritis include severe vertigo and dizziness, which subside in a few days. (Option C)

A 72-year-old man comes to the oncology clinic to discuss treatment options after the recent diagnosis of B-cell lymphoma; he is accompanied by his daughter. He reports that he is "fine but feels a little run down." His daughter reports that he is taking multiple naps daily. He has lost 9 kg (20 lb) over the last 6 months. He has fallen twice, without injury, in the last month. He does not use assistive devices to ambulate. Which of the following is the most appropriate referral for the next step in evaluation? (A) Physical therapy (B) Comprehensive geriatric assessment (CGA) (C) Psychiatry (D) Primary care evaluation

(B) Comprehensive geriatric assessment (CGA) CGA would be the most appropriate next step for this patient, given the several concerns that have arisen during the visit. CGA is an interprofessional process for identifying the medical, functional, cognitive, and psychosocial abilities and limitations of an older adult. The intent of the evaluation is to formulate a plan that focuses on a unifying diagnosis after interdisciplinary evaluation. The plan is then coordinated with the patient's primary care provider to facilitate adherence to recommendations. Indications for referral to CGA include advanced age, medical comorbidities, psychosocial disorders, specific geriatric conditions (eg, falls, cognitive decline, functional disability), predicted high health care utilization, and potential change in living conditions, (ie, level of care). The team consists of a physician, nurse, and social worker at its core, with an extended team that includes, as needed, physical and occupational therapy, pharmacy, psychology, psychiatry, nutrition, dentistry, optometry, podiatry, and audiology. Assessment focuses on functional capacity and quality of life and includes evaluation of cognition, polypharmacy, gait and balance (fall risk), social support, and goals of care. CGA differs from standard medical evaluation in that it includes nonmedical domains and involves an interprofessional team. Adherence to CGA recommendations leads to improved outcomes, including reduction in functional decline and improved care of falls and urinary incontinence (SOE=B). CGA is increasingly used for cancer patients to assess frailty and fitness to undergo treatment and to address goals of care. (Option B) This patient is likely to need physical therapy and assessment for possible depression, both of which would be included in CGA. Full psychiatric evaluation may be of value if problems are identified during CGA. It is unlikely that he would receive appropriate diagnoses without thorough evaluation of his cognition, function, and emotional state. In a primary care clinic, the extent of this patient's functional, psychosocial, and cognitive aspects of health may go undetected. (Options A, C, D)

A 74-year-old woman lives by herself in an apartment building for older adults. She has poor vision and urinary incontinence. She walks with a cane and has fallen three times in the past year. She has not seen a physician in the past 6 months, other than a visit to the emergency department after a fall. History includes hypertension, painful osteoarthritis, stroke, and frequent urinary tract infections. She takes ten prescription medications. Which of the following is most likely to reduce her functional decline and risk of nursing-home placement? (A) Protocol-based reduction in medications (B) Comprehensive geriatric assessment (CGA) with multiple home visits (C) Multimodality pain management (D) Physical therapy

(B) Comprehensive geriatric assessment (CGA) with multiple home visits CGA, coupled with an interprofessional home visit program addressing the needs identified by CGA, reduces functional decline and long-term use of nursing facilities (SOE=A). Further, the home visit program may reduce unnecessary emergency department visits and hospitalizations and may facilitate a home death for patients who prefer that option (SOE=B). (Option B) Medication reduction is generally a sound strategy in older adults, when it can be accomplished without destabilizing serious medical conditions, but it has not been shown to affect functional outcomes or nursing-home placement. (Option A) Pain management and physical therapy would both be important components of the overall comprehensive assessment and plan. Pain management using multiple modalities often improves symptom control in patients with chronic pain. (Option C) As part of a multifaceted intervention, physical therapy reduces the incidence of falls and is cost-effective in high-risk patients, mostly by preventing fractures. (Option D)

A 67-year-old woman comes to the office for a routine visit. History includes hypertension, atrial fibrillation, and hypothyroidism. Current medications are lisinopril 10 mg/d, levothyroxine 0.125 mg/d, and aspirin 81 mg/d. She is married and actively engaged in volunteer work and travel. She drinks 10 oz of wine daily with dinner, as she has for much of her adult life, and would like to continue this routine. Which one of the following is most consistent with US recommendations on alcohol use for this patient? (A) Stop consuming alcohol. (B) Decrease wine consumption to 5 oz daily. (C) Decrease wine consumption to 10 oz four days per week. (D) No change is necessary.

(B) Decrease wine consumption to 5 oz daily. In its 2020-2025 Dietary Guidelines for Americans, the US Department of Agriculture (USDA) advises adults to not drink alcohol at all, or to do so in moderation. It defines moderation for men as no more than two drinks on a day when alcohol is consumed, and for women as no more than one drink on a day when alcohol is consumed. One drink is the equivalent of 5 oz of wine, 12 oz of beer, or 1.5 oz of 80 proof distilled alcohol. The USDA provides the same recommendation for adults >60 years old yet notes that older adults are at higher risk of adverse effects from drinking. The National Institute on Alcohol Abuse and Alcoholism (NIAAA) recommends that adults ≥65 years old not consume more than three drinks in a single day or more than seven drinks in a week. The USPSTF recommends screening for alcohol use disorder in all adults ≥18 years old with a standard instrument, such as the Alcohol Use Disorders Identification Test-Consumption (AUDIT-C) or the Single Alcohol Screening Question (SASQ). The thresholds for alcohol misuse used by these instruments are consistent with the USDA recommendations. Significant debate remains regarding the safe level of alcohol consumption, and the USDA in its most recent guidelines recognizes that there is emerging evidence suggesting that drinking in moderation may have negative health effects. Many studies have found reduced risk of cardiovascular disease with moderate alcohol consumption compared with no consumption or heavy consumption. Some studies have found an association with increased quality of life with moderate consumption compared with no consumption or heavy consumption. However, studies of overall mortality have been mixed, and newer data suggest that risks of some cancers may be increased with even moderate alcohol consumption. This patient's current intake of 10 oz daily exceeds the weekly drinks recommendation for individuals >65 years old according to the NIAAA, so recommending no change is not appropriate. Limiting frequency to 4 days per week at her current daily intake would still involve consumption above the recommended weekly threshold. Eliminating alcohol entirely may be the healthiest choice. However, the patient has expressed a desire to continue alcohol consumption; decreasing her daily intake to 5 oz would be within the more conservative NIAAA recommendations for moderate consumption among older adults. (Options A, B, C, D)

An 86-year-old man comes to the office because he has hearing loss. He describes gradual bilateral hearing loss, with more difficulty hearing low-frequency than high-frequency sounds. He does not have tinnitus or dizziness. He has had no trauma to his ear, and he has never seen an audiologist or used amplification. On examination, the pinna is enlarged, and there are small amounts of ear wax. The tympanic membrane is pearly white, with no retraction. What is the most likely age-related physiologic change underlying this patient's symptoms? (A) Atrophy of the stria vascularis (B) Decreased efficiency of ossicular articulations (C) Decrease in size and number of otoliths (D) Loss of hair cells in the organ of Corti

(B) Decreased efficiency of ossicular articulations This patient's symptoms are consistent with conductive hearing loss, a middle ear disorder that frequently presents as difficulty hearing low-frequency sounds. Common causes include loss in the elasticity and efficiency of ossicular articulations and a thickened tympanic membrane. These changes are part of normal aging but can be exacerbated by conditions such as otosclerosis, infections, or trauma. (Option B) Age-related sensorineural hearing loss (presbycusis) is an inner ear disorder characterized by difficulty hearing high-frequency sounds. There are four types of presbycusis: sensory, metabolic, neural, and mechanical. When vibrations from the middle ear reach the cochlea, hair cells in the organ of Corti vibrate in response to different frequencies. Hair cells most sensitive to high-frequency sounds are encountered first, and over time they are more likely to atrophy than those that are most sensitive to lower frequency sounds. This preferential atrophy leads to high-frequency hearing loss. (Option D) The stria vascularis is the blood supply to the neurosensory receptors in the cochlea. Atrophy of the stria vascularis restricts blood supply to the neurosensory receptors, leading to symptoms of presbycusis. Neural presbycusis is due to loss of cochlear neurons in the basal part of the spiral ganglion; mechanical presbycusis is due to stiffening of the basilar membrane, which restricts motion of the cochlear duct. (Option A) The vestibular system, comprising the saccule, utricle, and ampulla, contains otoliths, which help with the perception of gravity, speed, and rotation. A decrease in size and number of otoliths does not lead to hearing impairment, but rather to symptoms of vertigo. (Option C)

A 92-year-old man comes to the office because his wife believes that his memory loss is compromising his ability to drive. He has not had any motor vehicle violations, and he thinks he is a good driver. History includes hypertension. Score is 22 of 30 on the Montreal Cognitive Assessment (MoCA); neuropsychologic testing is consistent with mild dementia. Which of the following is most appropriate? (A) Tell the patient that he should stop driving. (B) Refer the patient to the Department of Motor Vehicles (DMV) for a performance-based road test. (C) Monitor the patient's cognitive impairment every 6 months; report further impairment to the DMV. (D) Notify the DMV that the patient's driver's license must be revoked.

(B) Refer the patient to the Department of Motor Vehicles (DMV) for a performance-based road test. Some clinicians may be reluctant to refer patients for road tests because the procedures are rarely standardized and the data supporting their use may be limited (SOE=B). However, the ability to demonstrate proficiency behind the wheel in traffic is the method adopted by all 50 states to evaluate both novice and medically impaired patients. Therefore, referral for a performance-based road test is the most appropriate next step for this patient. (Option B) Because there is no common method of assessing dementia severity in relation to driving fitness, monitoring the patient's cognitive impairment every 6 months would not help to determine driving fitness. Rather, experts recommend that patients with mild dementia or with prominent impairments in key cognitive domains (eg, attention, visuospatial skills) be referred for a performance-based road test (SOE=B), as should patients whose caregivers observe them to be impaired. The DMV in every state conducts performance-based road tests. (Option C) An oral or written recommendation for this patient to stop driving, or a recommendation to the DMV that his license be revoked, is not an appropriate next step. There is evidence to suggest that drivers with dementia have at least a 2-fold greater risk of crashes than cognitively intact older adults, but this increased risk is not found in all studies. In addition, most traffic safety experts conclude that psychometric tests may serve to identify drivers at risk but should not be the sole determinant in decisions to revoke driving privileges. Most patients with mild dementia are able to pass a driving performance test (SOE=A). (Options A, D)

A 75-year-old woman comes to the office for consultation after she undergoes an audiology examination. Results show bilateral symmetric hearing loss sloping from mild to moderate at high frequencies. Bone conduction is equal to air conduction, and speech discrimination is 93%. On examination, both of her ears are free of cerumen and the tympanic membranes appear normal. Which type of hearing loss is this patient experiencing? (A) Conductive (B) Sensorineural (C) Mixed (D) Central auditory processing disorder

(B) Sensorineural This patient has sensorineural hearing loss: both air and bone conduction are depressed to the same degree and are at the same point on the audiogram. Sensorineural hearing loss is associated with presbycusis, tumors of the eighth cranial nerve, Ménière disease, ototoxicity, and noise exposure. High-frequency sensorineural hearing loss due to aging, noise exposure, or ototoxic agents is usually treated with hearing aids to enhance audibility of high-frequency, low-intensity consonants, which is key to understanding speech. Tinnitus and vertigo may accompany sensorineural hearing loss, or they may be symptoms of other medical conditions. (Option B) Conductive hearing loss is related to disorders in the external auditory canal or middle ear. In the external auditory canal, conductive hearing loss can be caused by impacted cerumen (common in older adults), otitis externa, exostoses, and congenital malformations. In the middle ear, causes for conductive hearing loss include effusion, otitis media, eustachian tube dysfunction, ossicular chain disruption, otosclerosis, and, rarely, benign or malignant tumors. The abnormalities obstruct the auditory signals being transmitted to the inner ear/cochlea. The hearing loss that results is categorized as conductive hearing loss. The prevalence of middle-ear abnormalities and hearing loss is higher among older adults with osteoarthritis and no family history of hearing loss, noise exposure, or chronic middle-ear effusion than among older adults without arthritis. (Option A) The audiogram, a graph of hearing level (loudness) as a function of frequency (pitch), describes the nature, severity, and configuration of an individual's hearing loss. The audiologist plots hearing level by frequency for right and left ears. The ability to hear tones transmitted by air and bone conduction is tested. Assessment of hearing by air conduction yields information about the status of the outer, middle, and inner ear; the eighth cranial nerve; and the central auditory pathways, including the brain. Assessment of hearing by bone conduction reveals the status of the cochlear (sensorineural) mechanism. If there is a difference between thresholds for air and bone conduction, with bone conduction normal or better than air conduction, the individual has a conductive hearing loss. Mixed hearing loss is diagnosed when hearing for air- and bone-conducted signals is depressed below normal, with air conduction significantly worse than bone conduction. In older adults with presbycusis, new onset of a middle-ear infection or impacted cerumen may temporarily cause mixed hearing loss. Mixed and conductive types of hearing loss can require referral to an otolaryngologist and are often reversible. (Option C) Central auditory processing disorders are caused by CNS diseases that interfere with ability to discriminate speech, especially when background noise is present. Causes include dementia, stroke, and presbycusis. Pure-tone audiometry is normal, but speech discrimination is impaired. This patient's speech discrimination is normal at 93%. When seen in combination with presbycusis, speech discrimination is substantially poorer than expected for deficits on pure-tone audiometry. (Option D)

A 66-year-old woman comes to the office for a routine visit. She has mild hypertension, which is controlled with diet, exercise, and regular blood pressure checks. She gets an annual flu shot and last received the Td booster at age 61. All findings on physical examination are unremarkable; blood pressure is 130/72 mmHg. In addition to the pneumococcal polysaccharide vaccine (PPSV23), which one of the following is recommended for her at this time? (A) Pneumococcal conjugate vaccine (PCV13) (B) Shingles vaccine (C) Td vaccine (D) Hepatitis B vaccine

(B) Shingles vaccine Shingles vaccine protects against shingles and its complications and is recommended for healthy adults ≥50 years old. For older adults, vaccination is highly efficacious in reducing risk of herpes zoster (~97%) and risk of post-herpetic neuralgia. Vaccination for herpes zoster can have an even more direct impact on older adults than influenza and pneumonia vaccinations. Herpes zoster infection is caused by reactivation of varicella zoster virus from a previous chicken pox infection. Its prevalence is approximately 50% in adults who are ≥85 years old; of patients ≥80 years old who are affected by herpes zoster, 15% have post-herpetic neuralgia and chronic pain. For patients who get shingles despite vaccination, Shingrix® decreases the risk of pain during and after infection and, for adults in an institutional setting, decreases the risk of being placed in isolation during active infection. (Option B) Tetanus is effectively controlled through vaccination every 10 years. Most of the relatively few cases of tetanus among older adults occur in under-vaccinated individuals. Because of the reemergence of whooping cough, acellular pertussis should be included with tetanus and diphtheria booster vaccination (Tdap). (Option C) Pneumococcal polysaccharide vaccine (PPSV23) protects against serious pneumococcal disease, including meningitis and bloodstream infections; it is recommended for all adults ≥65 years old and for vulnerable adults <65 years old. Pneumococcal conjugate vaccine (PCV13) protects against serious pneumococcal disease and pneumonia and is recommended for all adults with a condition that weakens the immune system and for adults who have a cerebrospinal fluid leak or a cochlear implant. For adults >65 years old, current Advisory Committee on Immunization Practices guidelines recommend discussion with the patient about risks and benefits from adding PCV13 to the immunization schedule. If it is added to the schedule, PCV13 should be administered first, followed by PPSV23 6-12 months later. (Option A) Hepatitis B vaccine is recommended for adults at increased risk, including health care workers and adults with chronic liver disease. (Option D)

A 65-year-old woman returns to the office for routine follow-up of high blood pressure and COPD. She mentions that her vision loss has been gradually getting worse; as a result, feeding herself has become more challenging. She has exudative age-related macular degeneration in her right eye and nonexudative age-related macular degeneration in her left eye. There is a central scotoma in both eyes. What is the most appropriate recommendation to improve her functional problem? (A) Use a magnifier to increase her field of view during meals. (B) Turn her head or move the plate to one side to use her peripheral vision during meals. (C) Use a plate guard and rocker spoon. (D) Enhance the color contrast between table ware and the table surface.

(B) Turn her head or move the plate to one side to use her peripheral vision during meals. Patients with low vision or blindness benefit from adaptive equipment to remain independent. Addressing the functional needs of affected patients also reduces their risk of depression. Low-vision services or clinics are often attached to ophthalmology and optometry clinics to assist patients in finding an appropriate device or approach for their problems and provide training. Patients are asked to bring a list of visual tasks that they have difficulty completing and that are not addressed by standard eye care providers. If a low-vision clinic is inaccessible, excellent resources are available online through the Lighthouse Guild (www.lighthouseguild.org) and the American Foundation for the Blind (www.afb.org). When central scotomas compromise activities of daily living (such as eating independently), use of preserved peripheral vision to view the target (eccentric viewing) is a helpful, accessible way to maintain independence. (Option B) Magnifiers are useful compensatory devices for reading and other near-vision tasks, but their use for eating would be cumbersome. Enhancing color contrast between the table and plate could also be helpful but would not allow this patient to see the food on the plate. Plate guards and rocker spoons are helpful for patients with tremor; they would not help this patient see the plate or food. (Options A, C, D)

A 73-year-old man has been having memory problems that have become more evident to his wife this year. In addition, he has had repeated unexplained falls. History includes hypertension, hyperlipidemia, diabetes, urinary incontinence, and constipation. Seven years ago, he began to have impaired smelling, altered taste, and fitful sleep with recurrent dream enactment; the dream enactments wake his wife at night. Symptoms progressed and fluctuating cognitive dysfunction and bilateral arm tremors developed. For several years, he has had hallucinations of children in the room. The hallucinations do not frighten him, but they worsened during hospitalization for urinary tract infection last year. Behavioral interventions for the patient's neuropsychiatric symptoms have been unsuccessful. MRI of the brain shows mild white matter changes. Which one of the following medications should be considered for this patient? (A) Citalopram (B) Clonazepam (C) Donepezil (D) Quetiapine

(C) Donepezil Lewy body dementia is the second most common dementia (after Alzheimer disease) and is likely underdiagnosed. It is notoriously difficult to diagnose before the onset of memory complaints. Subacute constipation, anosmia/hyposmia, and parasomnia are all associated with a higher risk of Lewy body dementia. Cardinal features are dementia, visual hallucinations, and extrapyramidal symptoms. Between 60% and 80% of patients have fluctuating cognition and alertness, including prominent daytime drowsiness, extended naps, and staring spells. Impairment in attention, executive function, and visuospatial skills is more prominent in Lewy body dementia than in other dementias. Well-formed or more abstract visual hallucinations occur in two-thirds of patients and vary in type. Extrapyramidal symptoms (bradykinesia, akinesia, rigidity, tremor, and abnormal gait) develop in 70%-90% of patients. Patients have prominent autonomic dysfunction, with urinary retention or incontinence, constipation, and impotence. Falls are especially common, likely due to a combination of orthostatic hypotension, extrapyramidal symptoms, and cognitive dysfunction. Treatment of Lewy body dementia is mostly symptomatic. Acetylcholinesterase inhibitors such as rivastigmine, galantamine, and donepezil have been shown to be effective in managing the cognitive and behavioral symptoms. Patients have severe neuroleptic sensitivity, and first-generation antipsychotics, such as haloperidol, should be avoided completely. Second-generation agents such as quetiapine also may cause severe extrapyramidal symptoms. (Options C, D) Benzodiazepines and melatonin have been used to mitigate parasomnias. Benzodiazepines such as clonazepam should be used cautiously, and benefits should be weighed against underlying risk of falls and confusion. Citalopram should only be considered after a cholinesterase inhibitor has been introduced. (Options A, B)

A 68-year-old man comes to the office because he is concerned that he is becoming forgetful. He states that his health is otherwise good. History includes mild osteoarthritis in his left knee. Medications include a multivitamin and occasional acetaminophen. He has begun to take an OTC sleep aid to help him sleep. His father, who had advanced Alzheimer disease, died 3 months ago, shortly after being moved to a nursing home. The patient says that he feels guilty that his father died alone. On review of systems, the patient admits to feeling tired but attributes this to his busy work schedule. Findings on physical examination are normal. Which of the following is the most appropriate next step? (A) Explain that minor forgetfulness is normal for his age. (B) Recommend that he stop the OTC sleep aid. (C) Refer for neuropsychologic testing. (D) Administer the Patient Health Questionnaire 9 (PHQ-9).

(D) Administer the Patient Health Questionnaire 9 (PHQ-9). It is natural for this patient to worry about memory loss after witnessing his father struggle with dementia. His symptoms of forgetfulness, fatigue, guilt, and insomnia could indicate depressed mood or a major depressive episode. The most appropriate next step is to screen for depression using an instrument such as the PHQ-9. (Option D) If depression screening is negative, cognitive testing would be a reasonable next step. There is some evidence that suggests subjective memory complaints are associated with cognitive decline (SOE=B). (Options A, C) It is important to identify this patient's OTC sleep aid. Many OTC sleep aids contain diphenhydramine, an anticholinergic medication that adversely affects cognition. The patient should stop using the sleep aid if it has anticholinergic effects. Melatonin, which is also available OTC, is an increasingly popular sleep aid. It does not have anticholinergic effects and is generally well tolerated; however, data supporting its efficacy is limited (SOE=C). Sleep disturbance is a common symptom of depression. Many patients with depression find that they sleep better once their depression is treated. (Option B)

A 65-year-old man has problems parking his van too close to the garage wall on the right. Recently, while backing out his vehicle, he hit the door and damaged the vehicle and garage. He has driven a truck for the past 50 year with no incident. His wife schedules an appointment with his ophthalmologist, whom he has not seen for 4 years. He sees his internist regularly for treatment of hypertension and diabetes. Which of the following is the most likely diagnosis? (A) Cataracts (B) Stroke (C) Glaucoma (D) Macular degeneration

(C) Glaucoma Primary open-angle glaucoma slowly causes restriction of the peripheral visual fields due to optic neuropathy without overt symptoms. Chronic optic neuropathy causes the loss of peripheral vision; it is often associated with increased intraocular pressure, although increased pressure need not be present for the damage to occur. In this patient, progressive glaucoma has restricted his peripheral visual field such that he is unable to see the garage wall (SOE=A). (Option C) Macular degeneration causes loss of central vision but does not impair peripheral vision. Rather than hitting the garage wall, a patient with macular degeneration might have difficulty reading a road sign in front of the car. (Option D) Cataracts cause blurring of vision, light sensitivity, and glare. Patients with cataracts often have difficulty driving at night. Visual impairment from cataracts usually progresses slowly. Loss of peripheral vision is unlikely to be an early symptom. (Option A) Stroke occurring behind the optic chiasm can cause bilateral hemispheric or quadrantanopic visual field loss. The deficit will be in the nasal visual field in one eye and the temporal visual field in the other. For the patient in this case, visual field loss seems to be monocular, not binocular, and thus less likely due to stroke. (Option B)

An 81-year-old man has had pain in his right calf for 6 months. Initially, he had pain after walking about one-quarter mile; now he has pain after walking 100 ft or with prolonged standing. The pain resolves completely when he sits. He has no pain in his back or upper leg. On examination, dorsalis pedis and posterior tibial pulses are 2+ bilaterally. Hair growth is good on both lower legs. There is mild immobility of the lumbar spine. Straight-leg raise tests are normal bilaterally. He has mild weakness of the right great toe extensor, right hip abductor, and right hip extensor. Radiography of the lumbar spine shows diffuse degenerative disc changes throughout the lumbar region. Which of the following treatments is most likely to be effective? (A) Angioplasty of lower leg vessels (B) Epidural corticosteroid injections (C) Laminectomy (D) Physical therapy

(C) Laminectomy This patient's pain is typical of the pseudoclaudication syndrome associated with lumbar spinal stenosis. The pain often involves only a portion of the affected leg, occurs when the patient stands and walks, and is relieved by sitting. The weakness of the L4-5 and L5-S1 innervated muscles of the right leg further suggests this diagnosis. Surgical laminectomy is currently the only well-validated treatment for lumbar spinal stenosis. It was more effective than conservative management in a trial that followed patients for 4 years (number needed to treat: 5) (SOE=B). (Option C) Whereas arterial insufficiency of the lower leg can cause a similar syndrome, it is unlikely given this patient's normal pulses and good hair growth on the legs. Although epidural corticosteroid injections are used extensively to treat lumbar spinal stenosis, a review of controlled trials did not demonstrate efficacy (SOE=B). Physical therapy is often recommended for patients with back pain, yet there are no randomized controlled trials that demonstrate its effectiveness in pseudoclaudication syndrome. Lumbar interspinous spacers have been used to treat neurogenic claudication in patients with lumbar spinal stenosis. Further studies are needed to clarify which patients would benefit from spacers. (Options A, B, D)

A 68-year-old man is brought to the office by his son for follow-up evaluation. The patient is a widower who lives alone. History includes type 2 diabetes mellitus, peripheral vascular disease, ischemic cardiomyopathy, and macular degeneration. A visiting nurse comes weekly and his son and daughter-in-law help him with shopping and chores at home. He has been hospitalized three times over the past 2 months, most recently for cellulitis of his right foot. The patient drove until 2 months ago, when his first hospitalization occurred. He also gave his checkbook to his son, "just for a little while, until I get better." The son voices concern about the father's ability to live independently. The patient becomes visibly upset and states that he will never leave his home. He suggests that his son is after his Social Security money. Which of the following is the most appropriate next step? (A) Evaluate for cognitive impairment. (B) Evaluate for possible financial exploitation and for mistreatment or abuse. (C) Perform a comprehensive geriatric assessment (CGA). (D) Arrange for a daily home health aide.

(C) Perform a comprehensive geriatric assessment (CGA). This patient has had a functional decline and is at high risk of rehospitalization. Because his physical, mental, and social statuses have changed, CGA is the most appropriate next step. Assessment includes functional, psychosocial, cognitive, economic, and environmental history, thorough medical evaluation, and discussion of goals of care. The assessment can be done at home or in an office, hospital, or nursing facility. Most comprehensive geriatric assessments are performed in the outpatient setting and may require more than one visit. An interprofessional team (including a geriatrician, nurse, social worker, pharmacist, and physical and occupational therapists, among others) works closely with the patient and family or other care provider. (Option C) Several red flags are raised during this visit, including the patient's ability to manage finances, his concern about financial exploitation, and his recent hospitalizations, as well as the son's experience of caregiver stress and his access to resources and support. Focusing on only some aspects of the patient's well-being—such as financial abilities and cognitive assessment—may miss other important issues that should be addressed. (Options A, B, D) Two Cochrane reviews have evaluated the evidence for CGA (SOE=A). In surgical inpatients, there was no statistically significant benefit in length of stay, readmissions, major complications, or rates of delirium. One randomized controlled trial showed a mortality benefit in patients admitted with hip fracture. In the review of studies in medical and surgical inpatients, comprehensive geriatric assessment was associated with an increased likelihood of survival and discharge home and with decreased likelihood of nursing home admission 3-12 months after discharge, but it did not improve function or cognition. In a 2019 randomized controlled trial of CGA in frail outpatients, there was a statistically significant decrease in mortality and frailty in the intervention group.

An 81-year-old Navajo woman comes to the office to establish care. She is accompanied by her daughter. She lives by herself in a remote area of the Navajo Nation, in a traditional Navajo home with no electricity, running water, or indoor toilet. She did not attend school and speaks limited English. Her daughter has been trying to convince her mother to move into her home, which has electricity and running water, but the patient refuses. History includes hypertension, osteoarthritis, and prediabetes. What information would be most useful in determining whether the patient can continue to live safely in her home? (A) Results of Montreal Cognitive Assessment test (B) Possession of a cell phone or medical alert device (C) Performance on the 5x Sit-to-Stand test (D) Ability to manage finances independently

(C) Performance on the 5x Sit-to-Stand test This patient lives in a hogan, a traditional Navajo dwelling. Without running water or an indoor toilet, she will need to be able to safely walk outside to use an outhouse. She may also need to haul water and firewood. The Five Times Sit-to-Stand test will give important information about her mobility and her risk of falling (SOE=A). (Option C) Any comprehensive geriatric assessment must take into account the patient's language and cultural background. Communication should be facilitated by a medical interpreter, rather than a family member. Assessment tools can be helpful, but adaptations may be needed, because the standard tools usually have not been validated in minority populations. Cognitive impairment should be assessed, but use of a standard test, such as the Montreal Cognitive Assessment, is unlikely to be helpful. The patient may have a falsely low score because of her lack of English literacy and formal education. A low-literacy screening tool and careful history from her daughter would be more helpful. (Option A) Patients in rural areas may live miles away from neighbors, not close enough to be heard if they call for help. They should be asked about their plans for an emergency. While a cell phone or medical alert device may benefit this patient, it does not in itself provide information about the safety of her current living situation. (Option B) The patient's ability to perform instrumental activities of daily living (IADLs) is important information; particularly important are changes in her ability to perform IADLs, which could be indicative of cognitive decline. If this patient does not manage her finances independently, it would be helpful to find out whether this is a recent change, or if she has always required assistance because of a language barrier or innumeracy. (Option D)

A 66-year-old man comes to the clinic for follow-up. History includes type 2 diabetes mellitus and stage 3 chronic kidney disease; 7 years ago he had an episode of shingles. Current medications include metformin, lisinopril, and aspirin. He received the live attenuated zoster vaccine 5 years ago. What is the most appropriate recommendation regarding the zoster vaccine for this patient? (A) Live attenuated zoster vaccine now (B) No vaccine (C) Recombinant adjuvanted zoster vaccine now (D) He is immune to zoster.

(C) Recombinant adjuvanted zoster vaccine now Herpes zoster affects approximately 1 in 1 million persons in the US each year, and the incidence increases with age. Two zoster vaccines are available currently: live attenuated vaccine (Zostavax®) and recombinant adjuvanted vaccine (SHINGRIX). There are no head-to-head comparison trials between the two vaccines. The CDC recommends preferential use of the recombinant vaccine over the live attenuated vaccine. In separate randomized controlled trials, recombinant vaccine had higher and longer-lasting efficacy for prevention of herpes zoster and post-herpetic neuralgia (SOE=A). Two randomized controlled trials of recombinant vaccine showed efficacy of 96%-97% for patients 50-69 years old, and efficacy of 91% for patients >70 years old. Recombinant vaccine is recommended for all persons >50 years old, including immunocompromised patients, regardless of prior receipt of live attenuated vaccine or history of shingles. Two doses, given 2-6 months apart, are necessary to achieve the efficacy shown in studies.

An 85-year-old woman comes to the office to establish care. History includes hypertension, mild osteoarthritis, and gout. Although she is concerned about getting shingles, she is wary of getting the Shingrix® vaccine today. Which of the following would preclude vaccination with Shingrix® today? (A) She is >80 years old. (B) She regularly babysits for her infant granddaughter. (C) She tested negative for immunity to varicella. (D) She received Zostavax 3 years ago.

(C) She tested negative for immunity to varicella. Recombinant zoster vaccine (Shingrix®) is recommended by the Advisory Committee on Immunization Practices (ACIP) for adults ≥50 years old. Studies have confirmed the safety of Shingrix® vaccination administered ≥5 years after vaccination with the live shingles vaccine used from 2006-2020 (Zostavax®), and there are no theoretical or data concerns to indicate that Shingrix® would be less safe or effective if administered <5 years after a patient received Zostavax®. A shorter interval may be acceptable for older adults: in the shingles prevention study, the efficacy of Zostavax was 18% at 3 years after vaccination (average follow-up time) for adults >80 years old at the time of vaccination. A subsequent long-term effectiveness study showed that vaccine efficacy waned over time in all age groups. Therefore, giving Shingrix now would be prudent. (Option D) Unlike Zostavax®, the efficacy of Shingrix® continues to be high (>90%) at advanced ages. Thus, this patient's age is not a contraindication. Adults with a minor acute illness, such as a cold, can receive Shingrix®. Adults with a moderate or severe acute illness should usually wait until they recover before getting the vaccine. This includes anyone with a temperature ≥38.5°C (101.3°F). It is safe for inoculated persons to be around infants, pregnant women, and immunocompromised individuals after receiving Shingrix®, because it is not a live vaccine and transmission of the virus after vaccination is not possible. (Options A, B) Even if this patient does not recall whether she had chicken pox as a child, serologic testing for varicella immunity is not recommended before administering Shingrix®. More than 99% of people worldwide >50 years old have been exposed to varicella zoster virus. The ACIP recommends that anyone born in the US prior to 1980 should be considered immune to varicella. However, if an individual does have a serologic test for immunity and it is negative, it is recommended to follow ACIP guidance for immunization against varicella, because Shingrix® has not been tested in persons seronegative for varicella and it is not indicated for prevention of varicella. (Option C)

An 86-year-old man is hospitalized because he fell and sustained multiple right-sided rib fractures and facial bruising without fracture. He is widowed and has two children who live out of state. History includes hypertension and mild COPD; he has a 30 pack-year smoking history. At baseline, he takes amlodipine 10 mg/d. He does not use inhalers or alcohol. He does not see a health care provider regularly. The trauma team plans to discharge him to a subacute rehabilitation facility, with the goal of returning to independent living once his physical rehabilitation is complete. On physical examination, vital signs are stable. Oxygen saturation is 98% on room air. BMI is 22 kg/m2. There is bitemporal atrophy, mild sarcopenia, and bruising on the periorbital area and right cheek. Lungs sounds are slightly diminished. Geriatric assessment is performed. He has poor social supports at home, and he appears frail, with a sad affect. Score on St. Louis University Mental Status (SLUMS) is 23 of 30 and he is alert and attentive. Score on Geriatric Depression Scale is 7 of 30, consistent with mild depression. Which of the following is his greatest risk factor for cognitive decline in the next few years? (A) Alcohol abstinence (B) Lack of routine medical care (C) Social isolation and loneliness (D) Recent fall

(C) Social isolation and loneliness With rising single-member households, increasing geographic and career mobility, and erosion of intergenerational living arrangements, loneliness is a rampant problem in Western societies. At any given time, up to one-third of older adults experience loneliness. Loneliness is a subjective negative feeling associated with a perceived lack of a wider social network (social loneliness) or absence of a specific desired companion (emotional loneliness). Generally, risks of loneliness and social isolation are poverty, poor health, decreased mobility, and family dispersal. Both social isolation and emotional loneliness can contribute to worse outcomes such as cardiovascular disease, dementia, and higher risk of nursing home placement. Loneliness and depression in older adults are strongly associated. According to a 2017 study based on data derived from the Health and Retirement Study (a large cohort of older adults), the estimated rate of cognitive decline is approximately 20% faster in lonely adults than in adults who do not experience loneliness. (Option C) Lack of routine medical care and falls pose a risk for cognitive decline, but not as much as social isolation. The association between abstinence from alcohol and cognitive decline is controversial. (Options A, B, D)

A 69-year-old man comes to the office for routine follow-up. He is an established patient who is a recently retired executive in the airline industry. History includes type 2 diabetes mellitus. Medications are metformin 1,000 mg twice daily and lisinopril 2.5 mg/d for renal protection. He manages all basic and instrumental activities of daily living. He exercises for 30 minutes once or twice per week. He does not smoke; alcohol consumption is two drinks per day and two to three per day on weekends. He is actively engaged in the lives of his three children and seven grandchildren. He has noticed some mild cognitive lapses recently, such as difficulty with word finding, and is now concerned about how to maintain his cognitive health as long as possible. His most recent hemoglobin A1c was 7%. Systolic blood pressure is usually 120-130 mmHg in the office. Which of his current practices will be most likely to help him maintain cognitive health? (A) Alcohol consumption (B) Exercise program (C) Social network (D) Metformin use

(C) Social network The World Health Organization recommends seven lifestyle interventions to prevent cognitive decline: 150 minutes of moderate-intensity aerobic physical activity per week, or 75 minutes of vigorous-intensity activity per week Quitting smoking Nutritional measures that focus on plant-based diets (Mediterranean diet, MIND diet) or intermittent fasting. Stop harmful drinking. Cognitive training (a series of tasks designed to improve particular cognitive function) Social engagement Maintaining optimal weight This patient's strong social network is the lifestyle factor most in his favor for maintaining his cognitive health. His alcohol consumption exceeds the recommended amount and meets criteria for at-risk drinking; he should be advised to reduce intake to no more than one drink/day. His current exercise regimen does not meet the WHO recommendation for preventing cognitive decline; he should be encouraged to increase exercise to the recommended level. (Options A, B, C) Metformin is touted as an antiaging compound but proof has yet to emerge. A 2-year (2009-2011), double-blind, randomized controlled trial of 2,654 individuals, the Finnish Geriatric Intervention Study to Prevent Cognitive Impairment and Disability (FINGER), looked at multiple domains (nutritional guidance, exercise, cognitive training, social activity, and management of metabolic and vascular factors). Findings from this trial suggest that a multidomain intervention could improve or maintain cognitive function. (Option D)

A 78-year-old man comes to the office for a follow-up visit, accompanied by his wife. Possible Alzheimer disease was diagnosed 8 months ago, at which time CT of the brain showed mild atrophy and moderately severe periventricular and subcortical white matter microvascular disease. Medications include transdermal rivastigmine, aspirin 81 mg/d, and simvastatin. At today's visit, his score on the Montreal Cognitive Assessment (MoCA) is 23 of 30, and his Clinical Dementia Rating (CDR®) is 1. At the end of the visit, the patient's wife expresses concern about his ability to drive safely. She sometimes feels nervous when her husband is behind the wheel. She notes that her husband has stopped driving at night or when it rains, that he drives shorter distances and less often since he retired 5 years ago, and that he received a traffic citation about 4 years ago but has had no accidents or additional citations since then. When asked if he thinks he is a safe driver, the patient replies, "Well, I'm probably a little bit slower than I used to be, but overall, I'd say yes, I'm still a perfectly safe driver." Which of the following is the strongest evidence of the patient's risk of unsafe driving? (A) Patient's self-restriction and situational avoidance (B) CDR® score of 1 (C) Spouse's concern (D) Score ≤24 on MoCA

(C) Spouse's concern The US is an automobile-dependent society with a demographically expanding population of older adults and an increasing prevalence of dementia. Driving can be an important component of an older adult's independence and self-esteem. In patients with dementia, progressive cognitive decline coupled with poor awareness of acquired deficits, impaired judgment, or both, presents a significant public health challenge to driving safety. In 2010, the American Academy of Neurology updated practice parameters for evaluation and management of driving risk in patients with dementia. The parameters were based on systematic review of the strength of evidence for patient demographics, driving history, and cognitive tests to predict driving capability among patients with dementia. Ratings of "useful," "not useful," or "insufficient evidence" were applied to 12 characteristics, along with the level of evidence (strong, good, or weak) supporting the rating. There is good evidence to support use of the spouse's rating of a patient's driving ability to identify patients at increased risk of unsafe driving (SOE=B). (Option C) The evidence is weak regarding the usefulness of elements of the driving history—such as driving fewer miles, situational avoidance (eg, not driving at night or in the rain), and history of traffic citations or crashes—to identify patients at increased risk of unsafe driving (SOE=C). MoCA scores <18 may suggest an individual is at risk of unsafe driving. Evidence is strong to establish the CDR® scale as a global measure useful for identifying patients at increased risk of unsafe driving; those who score ≥2 should not drive (SOE=A). (Options B, D) Evidence is strong that a patient's self-rating of driving ability is not useful in identifying increased risk of unsafe driving (SOE=A). (Option A) Evidence regarding the benefit of comprehensive neuropsychologic testing to establish risk of unsafe driving is insufficient. The AGS and American Medical Association have published a functional assessment battery for older drivers (including those who do not have dementia) that includes tests of visuospatial cognitive ability and attention.

A 68-year-old man with Parkinson disease is brought to the clinic by his wife because of his physically aggressive behavior. She provides a synopsis of the disease progression: 15 years ago, he started to have vivid dreams during which he would kick out; 5 years ago, he started to have problems with walking and postural stability, leading to several ground-level falls with no major injuries; 4 years ago, Parkinson disease was diagnosed; 3 years ago, the patient started to believe that someone else was in the room when he was actually alone. In the past year, he is increasingly convinced that his wife has been replaced by an identical double, whom he pushes in frustration, hoping to get his "wife" back. He now uses a walker at home and a wheelchair outside. He no longer drives, and he has started to have trouble finding his way around the house. He sometimes forgets recent events, but he recalls them easily when cued by his wife. His wife took over money management recently as he was making too many errors, which she attributes to his intermittent "spacing out". Current medications are levodopa, polyethylene glycol, melatonin, and quetiapine. There is no additional past psychiatric or neurologic history, nor is there a history of substance use. On physical examination, there is marked axial rigidity with minimal rest tremor. He scores 24/30 on the Mini-Mental State Examination (MMSE): 4 of 5 points are lost for serial 7's, 1 of 3 points for the multi-step task, and 1 point for inability to copy intersecting pentagons. Which of this patient's symptoms are characteristic of Parkinson's psychosis-dementia complex? (A) Extrapyramidal symptoms dominated by tremor. (B) Free memory recall that is not improved with cueing. (C) The presence of minor hallucinations (D) Relatively preserved visuospatial skills

(C) The presence of minor hallucinations A psychosis-dementia continuum of disease has been reported in patients with Parkinson disease. Around 90% of patients with Parkinson disease and dementia have ≥1 neurobehavioral symptoms (SOE=A). Patients with more severe dementia and more advanced Parkinson disease have more neurobehavioral symptoms; the most severe dementia and most advanced motor symptoms are associated with psychosis and agitation. The most common psychotic phenomena in Parkinson disease are minor hallucinations. (Option C) These involve the sense of someone being present in the room when there is no one there; sideways passage (fleeting images in the peripheral visual field, sense of someone having passed by who was not clearly visible); and illusions (misperception of a real stimulus, usually visual) (SOE=A). The motor subtypes in Parkinson disease are postural instability and gait disorder (PIGD) and tremor-dominant (TD) phenotypes (SOE=A). The PIGD subtype has a more malignant course, with rapid progression and association with a higher incidence of cognitive decline, apathy, and depression (SOE=A). The PIGD phenotype has been found to be more common in patients with both major and minor psychotic symptoms as compared with patients who have no psychosis (SOE=B) (Option B). This patient's symptoms and pattern of progression are well within the psychosis-dementia spectrum. He first started to experience REM sleep behavior disorder, followed by minor (presence) hallucinations in the context of a PIGD-type of Parkinson disease, followed by a major psychosis (ie, full-blown Capgras syndrome), followed by cognitive symptoms. Regarding the pattern of cognitive impairment in PD dementia, the diagnostic criteria include two core features—a pre-existing diagnosis of Parkinson disease and a dementia syndrome defined by the presence of cognitive decline from premorbid baseline affecting >1 cognitive domain—and the dementia should contribute to functional decline independently of any decline due to the motor or autonomic features of Parkinson disease. This patient meets criteria for dementia. Typical associated features include impaired attention that may fluctuate, and impaired executive functions, visuospatial skills (Option D), and free recall memory that improves with cueing (Option B), with or without ≥1behavioral symptom. This patient is now experiencing decline in >1 cognitive domain, with impaired attention that fluctuates ("spacing out" symptom), and visuospatial impairment, resulting in problems with money management and finding his way around the home. (Option C) Impaired delayed recall that improves with cueing is typical (but not required) of Parkinson disease-related cognitive decline, which is in contrast to the amnestic syndrome of the hippocampal type that is seen in Alzheimer disease, where the free recall deficit is not normalized by cueing (SOE=A). (Option B) Two levels of diagnostic testing have been recommended for PD dementia by the Movement Disorder Society task force to operationalize the diagnosis of PD dementia, with Level I being appropriate for office screening by clinicians who have no neuropsychologic expertise, and Level II involving neuropsychologic testing that is more suitable for research. Full neuropsychologic testing or an occupational therapy assessment is therefore not necessary to establish a diagnosis of dementia for this patient in the clinic. However, the MMSE has not been found to be a suitable global rating scale for cognition in Parkinson disease by the International Parkinson and MDS Committee on Rating Scales Development and should not be used for this purpose (SOE=A); even its modified version for patients with Parkinson disease is only recommended with caveats. (Options A, D)

A 78-year-old woman comes to the office to establish care. She has no available medical records. During review of systems she acknowledges feeling tired, with low energy much of the time, and she is prone to falling asleep if she sits quietly. She has no appetite and does not sleep well at night. She also acknowledges that she has noticed frequent, small, unformed stools. History includes partial colectomy for colon cancer about 6 years ago. She says that she did not receive chemotherapy at the time and has had no follow-up colonoscopy since the surgery. Which of the following is the most appropriate next diagnostic step? (A) Fecal occult blood samples × 3 (B) Fecal immunochemical screening test (FIT) (C) CT colonography (D) Colonoscopy

(D) Colonoscopy This patient is a colon cancer survivor. She should have had surveillance colonoscopy at 1 year after surgery and then every 1-3 years, depending on prior findings (SOE=B). Given that she has not had this surveillance, that she is fatigued, and that she has a change in stools, colonoscopy should be offered to exclude recurrence. The importance of surveillance should be emphasized, and obstacles identified. Discussion should include preferences regarding treatment if recurrence is detected. Colonoscopy should proceed only if the patient is fit enough to tolerate treatment and wishes to pursue treatment if cancer is detected. (Option D) Regarding colon cancer screening, screening colonoscopy for patients with a remaining life expectancy of <10 years is not recommended because of the decreasing benefit due to increase in competing risks of death. The time-to-benefit for any improvement in survival associated with colon cancer screening is estimated to be ≥5 years, based on evidence from randomized controlled trials (SOE=A). This benefit must be weighed against the burdens and risks of each modality. Approximately 4 in 10,000 colonoscopies are complicated by perforation, and 8 in 10,000 complicated by major bleeding. The rates of complications rise with age. Endoscopic colonoscopy remains the gold standard for screening and surveillance, although newer technologies—including CT colonography and FIT—are alternative options for screening. Fecal occult blood testing is inadequate and not recommended for surveillance after colon cancer diagnosis. (Options A, B, C)

A 75-year-old man returns to the office after consultation with an audiologist. He received a diagnosis of age-related hearing loss and wants advice regarding interventions. He has difficulty understanding speech at meetings and in restaurants; he "hears people talking but can't make out the words." His score on the Hearing Handicap Inventory for the Elderly short form (HHIE-S), a self-reported questionnaire that quantifies the handicap. Pure-tone test results indicated a hearing loss sloping from mild to moderate, most pronounced in the high frequencies. Speech understanding in a quiet environment was excellent but decreased to fair in a noisy environment. Which one of the following is the most suitable intervention for this patient? (A) Personal sound amplification system (B) Bone-anchored hearing aid (C) Assistive listening technology for use in noisy environments (D) Digital hearing aid

(D) Digital hearing aid Presbycusis, or age-related hearing loss, is a multifaceted, degenerative condition that affects >10 million people globally. The primary sites of degenerative changes include the stria vascularis, sensory hair cells, and spiral ganglion neurons (SOE=B). Degenerative changes also occur throughout the brainstem into the temporal lobe of the auditory cortex. Modifiable risk factors associated with age-related hearing loss include noise exposure, diet, ototoxic medications, and smoking. Nonmodifiable risk factors include increasing age, sex, and race (SOE=B). Comorbidities associated with presbycusis include hypertension and cerebrovascular disease. This patient has mild-to-moderate age-related hearing loss, which is not amenable to medical or surgical treatment; hence, a digital hearing aid is the treatment of choice. In the US, only 14% of individuals with hearing loss actually use hearing aids (SOE=B). Use increases with age (from 4.3% among adults 50-59 years old, to 22% among adults ≥80 years old) (SOE=B). (Option D) Preferred hearing aid models are digital and include feedback suppression and dual microphones to enhance the level of the speech signal relative to the background noise. There is growing evidence that hearing aids improve ability to function and health-related quality of life (SOE=B). One randomized controlled trial of hearing aid use found positive effects on functional outcomes, depression, and cognition (SOE=A). Until 2016, federal law restricted the sale of hearing aids to adults >18 years old who have obtained a medical evaluation from a licensed physician, preferably one specializing in diseases of the ear. A waiver is available for adults who decline medical evaluation for religious or personal beliefs that preclude consultation with a physician. In 2016 Congress passed legislation for direct-to-consumer sales of FDA-approved hearing aids, known as over-the-counter aids. Such devices are appropriate for individuals with mild to moderate loss and are expected to become available in 2021 or soon thereafter. Audiologists and providers of hearing aids are required to advise prospective patients of the following medical contraindications to hearing aid use: unilateral hearing loss of sudden or recent onset (within the previous 90 days); active drainage from the ears within the previous 90 days; acute or chronic dizziness; air-bone gap ≥15 dB at 500, 1000, and 2000 Hz; visible congenital or traumatic deformity of the ear; and pain or discomfort in the ear. These contraindications require referral to a physician. Nonmedical barriers to hearing aid use include sociodemographic factors, such as cost and stigma, and personality variables, such as motivation and readiness. Severity of hearing loss is rarely a contraindication for hearing aids. Studies show success with cochlear implants in individuals with moderate to severe hearing loss. Cochlear implants improve speech understanding to a greater degree than traditional hearing aids. Communication strategies should be adopted during discussion with older adults who have hearing loss, such as obtaining the listener's attention before speaking, facing the listener while speaking, and eliminating background noise. A bone-anchored hearing aid is a device that is surgically implanted in the mastoid bone. The implanted post connects to a speech processor that transforms sound waves into vibrations that are transmitted to the inner ear via the skull, thereby bypassing the middle ear. Bone-anchored hearing aids are helpful for individuals who have unilateral hearing loss, conductive or mixed hearing loss involving the middle ear, or a malformation that precludes use of a hearing aid. (Option B) Personal sound amplification systems are intended for individuals with mild hearing loss. The sound quality and safety of these devices vary greatly. (Option A) Assistive listening technology is designed to augment digital hearing aids or cochlear implants when there are residual hearing deficits. This patient should first be fit with a digital hearing aid, and residual hearing deficits could then be resolved with assistive listening technology.(Option C)

An 89-year-old man comes to the office for a routine visit. History includes peripheral vascular disease. He smoked cigarettes, two packs per day for 20 years, until age 40. Which one of the following is the most appropriate screening interval for abdominal aortic aneurysm (AAA) for this patient? (A) Annually (B) Biannually (C) Screen once (D) Do not screen

(D) Do not screen An AAA is a weakening in the wall of the infrarenal aorta that results in an anteroposterior diameter ≥3 cm. It is found in approximately 4%-8% of older men. Approximately one-third of aneurysms will rupture. The US Preventive Services Task Force recommends one-time screening for men 65-75 years old who have ever smoked (SOE=A). There is no evidence to support screening for men >75 years old, regardless of smoking history. (Options A, B, C, D) AAAs are often undiagnosed; a large proportion are asymptomatic until they rupture. The rupture is generally acute and most often fatal (59%-83% of patients die before hospitalization). Significant risk factors for AAA include advanced age, male sex, smoking, and family history of AAAs. Other risk factors include history of other vascular aneurysms, coronary artery disease, cerebrovascular disease, atherosclerosis, hypercholesterolemia, and hypertension. Protective factors include Black race, female sex, and presence of diabetes mellitus. Ultrasonography is the standard screening method for AAA because it has high sensitivity (94%-100%) and specificity (98%-100%). It is noninvasive and relatively inexpensive, and it does not involve exposure to radiation.

An 80-year-old woman comes to the office because the vision in her right eye has suddenly become blurred. The external appearance of the eye is normal. The right pupil does not constrict when a flashlight is shined on it. When the flashlight is swung to the left eye, the right pupil constricts, and when it is swung back to the right eye, the right pupil dilates. Which one of the following is the most likely explanation for these findings? (A) Right occipital lobe infarct (B) Left occipital lobe infarct (C) Iritis of the right eye (D) Ischemic optic neuropathy

(D) Ischemic optic neuropathy The swinging flashlight test is a simple, inexpensive, and reproducible bedside technique for detecting a relative afferent pupillary defect (SOE=B). In this patient, the light shining on the normal left eye results in bilateral constriction of the pupils, but the abnormal right pupil dilates when the normal left eye is no longer responding to the light. A positive test indicates disease in the retina or optic nerve. The test would not be positive if visual loss were caused by an occipital lobe infarct because the pupillary light reflex uses an entirely subcortical pathway. In iritis, a ciliary flush would be seen with injection around the limbus on inspection. Also, the pupil would be small and would not dilate in response to the swinging flashlight. (Options A, B, C, D)

An 85-year-old woman has had recent episodes of dizziness in which she senses the room spinning around her, feels her right ear is blocked, and hears a roaring sensation. The symptoms improve gradually until she returns to baseline over a few hours. She has no associated headaches. History includes diabetes and hypertension. Which of the following is the most likely diagnosis? (A) Benign paroxysmal positional vertigo (BPPV) (B) Acute labyrinthitis (C) Transient ischemic attack (D) Ménière disease

(D) Ménière disease Several characteristics, including duration of vertigo and presence of aural symptoms, help establish the diagnosis of Ménière disease in this patient. Episodes of Ménière disease last several hours and include the triad of vertigo, hearing loss (commonly low frequency, which may be subjectively described as blocked ears), and tinnitus (often roaring). The pathophysiology of Ménière disease is believed to be related to pressure from excess endolymphatic fluid leading to inner ear dysfunction (SOE=B). (Option D) BPPV is the most common cause of vertigo; its underlying pathophysiology is thought to be displacement of canaliths from the utricle into the semicircular canals, primarily the posterior semicircular canal (SOE=B). Patients with this disorder have episodic vertigo lasting no more than a few minutes and no other associated otologic symptoms. (Option A) Patients with symptoms lasting >2 days are likely to have either vestibular neuritis or acute labyrinthitis. The distinguishing characteristic between these otogenic sources of true vertigo is hearing loss. Labyrinthitis involves inflammation of the inner labyrinthine structures where cochlear and vestibular apparatuses are in continuity, causing both hearing loss and vertigo. In contrast, vestibular neuritis is believed to be associated with inflammation of the vestibular nerve, thus sparing the cochlea. (Option B) In this case, transient ischemic attack is unlikely, given the recurrent nature of the vertigo episodes, lack of brainstem symptoms, and the accompaniment of hearing loss and tinnitus. (Option C)

Several epidemiologic studies describe an association between hearing impairment and dementia. Many hypotheses have been proposed to explain this association, and there is debate about whether there is a causal link between hearing loss and dementia. Which of the following is consistent with the hypothesis that hearing loss is unrelated to risk of dementia? (A) Cognitive load hypothesis (B) Common cause hypothesis (C) Cascade hypothesis (D) Overdiagnosis or harbinger hypothesis

(D) Overdiagnosis or harbinger hypothesis Many epidemiologic and basic science studies have demonstrated that hearing loss is an independent and modifiable risk factor for development of cognitive dysfunction. In a meta-analysis of three high-quality studies, the Lancet Commission on Dementia Prevention, Intervention, and Care identified a pooled relative risk of 1.94 (95% CI, 1.38-2.73) for development of cognitive decline in adults with age-related hearing loss. Several cohort studies have demonstrated that even mild levels of hearing loss increase the risk of cognitive decline over time. However, a causal relationship has yet to be elucidated. Further, hearing rehabilitation (via methods such as hearing aids and cochlear implantation) does not completely reverse comorbid cognitive dysfunction. The main hypotheses addressing the relationship between auditory and cognitive decline are 1) cognitive load hypothesis, 2) common cause hypothesis, 3) cascade hypothesis, and 4) overdiagnosis (harbinger) hypothesis. Cognitive load hypothesis postulates that hearing loss leads to greater use of cognitive resources during listening or auditory perceptual processing. Effortful listening leads to depletion of cognitive reserve, thereby diverting cognitive resources from other tasks. (Option A) Common cause hypothesis postulates that hearing loss and cognitive impairment do not cause one another, but instead result from a common neurodegenerative process in the aging brain. The etiology of age-related hearing loss is multifactorial and may result from degeneration of the stria vascularis, loss of hair cells and primary afferent neurons, as well as reduction in neurons in the central auditory pathways. (Option B) Cascade hypothesispostulates that hearing loss leads to several adverse downstream effects, resulting in cognitive decline. For example, hearing loss can lead to communication breakdown and social isolation. Research suggests that social isolation and depression are risk factors for poorer overall cognitive performance and faster cognitive decline.(Option C) Overdiagnosis or harbinger hypothesis posits that hearing dysfunction, rather than cognitive dysfunction, affects performance on certain cognitive tests. Thus, hearing loss itself could result in an overestimation of the level of cognitive dysfunction. (Option D)

An 84-year-old woman comes to the office for her annual examination. She lives alone in her own home and remains engaged with church and social groups. History includes osteoarthritis of the cervical and lumbar spine and mild hearing impairment. Current medications are acetaminophen 1,000 mg three times daily and calcium 600 mg plus vitamin D 400 IU twice daily. She states that her neck and back pain are well controlled with acetaminophen. She is concerned about two episodes that occurred when she was backing up her car. On one occasion, she dislodged the rearview mirror on the passenger side while backing out of her garage. On the second occasion, she put out the passenger-side taillight when she struck a fire hydrant. She believes it is important for her to continue driving. Functional assessment demonstrates intact cognition and vision, with near visual acuity 20/30 in each eye. The whisper test is positive for hearing impairment. There are no other sensory deficits. On physical examination, there is decreased range of motion in her neck and back in all planes, which is unchanged from last year's findings. There are diffuse osteoarthritic deformities in the knees and hands and decreased range of motion at the hips. There is mild kyphosis. Motor strength and deep tendon reflexes are symmetric and intact. Which one of the following is the most appropriate next step? (A) Recommend that she no longer drive. (B) Refer to physical therapy. (C) Refer to audiology. (D) Refer to occupational therapy.

(D) Refer to occupational therapy. For many older adults, driving remains a vital link to community, independence, mobility, and quality of life. A recommendation not to drive would have major quality-of-life implications for this socially engaged woman. Occupational therapists may improve functional abilities in both basic and instrumental activities of daily living. This patient should be referred to an occupational therapist with additional training and subspecialty certification in driver evaluation and treatment. An occupational therapist engaged in driver safety programs will be able to look both at the patient and her vehicle and provide adjustments or adaptive equipment (eg, better rearview mirrors) to help her compensate for deficits (SOE=C). (Options A, D) The patient's difficulty with range of motion in her neck and the reported accidents while backing up her car both suggest that her driving limitation is due to her neck disease. Exercises involving neck range of motion, as offered in physical therapy, are helpful in pain management but unlikely to improve range of motion sufficiently to compensate for her driving difficulties. (Option B) Although hearing loss is a risk factor for impaired driving, the mild nature of the patient's disorder and her major difficulty with cervical arthritis make audiology evaluation a much lower priority. (Option C)

An 80-year-old man comes to the office accompanied by his wife, who is concerned because he has been seeing mice and bugs crawling on the ceiling many nights in the past week. The hallucinations frighten him and prevent him from sleeping at times. History includes Alzheimer disease, hypertension, benign prostatic hyperplasia, vitamin D deficiency, and primary open-angle glaucoma that was newly diagnosed 1 month ago. Current medications are donepezil, amlodipine, tamsulosin, cholecalciferol, timolol, and latanoprost. Which of the following medications is most likely to be contributing to his hallucinations? (A) Donepezil (B) Amlodipine (C) Latanoprost (D) Timolol

(D) Timolol Systemic absorption of topical ocular medications can have unintended adverse effects, particularly if the eye drop is improperly administered. Proper administration of topical drops consists of instilling a single drop under the lower lid, closing the eye for at least 1 minute and applying gentle pressure on the lacrimal sac for 15-30 seconds. Many patients (and caregivers who assist in administering eye drops) are unaware of this procedure. Timolol is a nonselective β-blocker that can cross the blood-brain barrier. Cases of hallucinations have been reported with this medication in older adults. If hallucinations occur, timolol should be discontinued and follow-up is needed to assess for resolution. Additional adverse systemic effects of topical β-blockers include hypotension, bradycardia, heart failure, bronchospasm, lethargy, weakness, and anxiety. Other glaucoma interventions that may cause systemic adverse effects include α-agonists, α/β agonists, and miotic cholinesterase inhibitors. Ocular topical agents should be included in medication reviews when there are new symptoms, especially if the agent was recently added. (Option D) Donepezil has been used for the management of psychosis in Alzheimer disease, yet hallucinations develop with treatment in as many as 3% of persons. In the case presented here, timolol is the more likely cause of the hallucinations, given the temporal relationship between start of a new agent and onset of hallucinations. Hallucinations are not reported adverse effects of either latanoprost or amlodipine. There have been no reports of systemic effects in the class of prostaglandin analogues to which latanoprost belongs. (Options A, B, C) The combination of hallucinations and insomnia may lead to a prescribing cascade that should be avoided. Adding medications without first identifying the root cause of the problems exposes the patient to more medications and more potential adverse effects.

A 75-year-old woman comes to the office to establish care. History includes urge incontinence, seasonal allergies, and obesity (BMI 31 kg/m2). She reports gaining about 9 kg (20 lb) over the past 10 years. She exercises a few times a week by walking and using her home weights. She asks whether she should lose weight. Which of the following is the best response? (A) Even modest weight loss will reduce her mortality risk. (B) She should target weight loss to achieve BMI between 18 kg/m2 and 25 kg/m2 for optimal survival. (C) Weight loss is not necessary because ideal BMI for older adults may be between 30 kg/m2 and 35 kg/m2. (D) Weight loss will improve several important health outcomes but not mortality.

(D) Weight loss will improve several important health outcomes but not mortality. BMI <18.5 kg/m2 indicates that an adult is underweight; BMI between 18.5 kg/m2 and 24.9 kg/m2 is considered normal weight; and BMI between 25 kg/m2 and 29.9 kg/m2 indicates that an adult is overweight. Individuals with BMI ?30 kg/m2 are considered obese. Cut-off values are the same for all adults, irrespective of age. Because BMI is calculated using height and weight alone and does not adjust for muscle mass or body composition, it may be less useful for determining obesity in older adults. There is evidence from large cohort studies that being overweight or obese is associated with higher all-cause mortality (SOE=A). Among older adults, some studies have shown that being overweight or obese may be associated with lower mortality risk, the so-called "obesity paradox" (SOE=B). It is possible that obesity offers some health benefits in older adults, such as protection from loss of bone mass density and osteoporotic fractures, adequate energy reserve during negative energy balance, and preservation of muscle mass (SOE=C). Sarcopenic obesity (decreased lean mass with increase in body fat) is associated with an increased mortality rate (SOE=A). (Option A) The ideal BMI in older adults is unknown, and more evidence is needed from prospective clinical trials. Intentional weight loss in younger adults can reduce mortality and morbidity associated with obesity-related health problems (SOE=A), but no studies have shown a mortality benefit in older adults. Health outcomes that are often seen in older adults who lose weight include lower blood pressure and hemoglobin A1c. Physical fitness and functional independence appear to confer more benefit than weight loss in older adults. Some studies suggest that carefully monitored intentional weight loss in obese older adults can improve functional status (SOE=C). (Options B, C, D)

You are asked to evaluate a 75-year-old female with a history of coronary artery disease and hypertension who presents with forgetfulness and difficulty caring for herself. She is accompanied by her son who has noticed a progressive functional decline in his mother over at least the past year. She has been increasingly forgetful, including several incidents wherein she would leave water for tea boiling for hours. In the past two months, he has taken over the management of her bills, as she had missed several payments. She is able to ambulate without assistance. Two days ago, her neighbors found her wandering around the neighborhood in the middle of the night. When they tried to redirect her, she became agitated, yelling that strangers were trying to rob her. She calmed down when her son arrived at the scene. She states that she does not feel depressed. Her vital signs are normal. Her physical exam was unremarkable and a detailed neurological exam demonstrates no significant abnormal findings except for a Montreal Cognitive Assessment (MoCA) score of 10. She completes a "Get Up and Go" test in 15 seconds, rising from the chair and ambulating without difficulty. A complete blood count, thyroid stimulating hormone, and vitamin B12 level are normal. A non-contrast computed tomography (CT) of her head shows increased sulci. No other structural abnormalities are seen. What is the most-likely diagnosis? A. Alzheimer's dementia B. Frontotemporal dementia (FTD) C. Lewy body dementia D. Pseudodementia secondary to depression E. Vascular dementia

A. Alzheimer's dementia This patient likely has Alzheimer's dementia. She is demonstrating a gradual memory loss, short-term memory loss, and a progressive decline in function. While the diagnosis can only be confirmed at autopsy, a clinical diagnosis is 90% accurate. Vascular dementia is characterized by abrupt onset and step-wise progression of a patient's memory impairment often in the setting of a history of a stroke or with focal neurologic symptoms. Given her history of coronary artery disease and hypertension she has risk factors for this, but overall the history is more consistent with Alzheimer's​ dementia. Frontotemporal dementia is characterized by behavior and personality changes and language impairment. This can manifest as disinhibition, apathy, hyperorality, and compulsive behaviors such as hoarding or cleaning. This patient does not demonstrate this constellation of symptoms. Patients with Lewy body dementia typically have motor parkinsonism and visual hallucinations, this patient does not have these symptoms. In addition,​ no symptoms of depression are noted making pseudodementia secondary to depression unlikely.

A 75-year old man has gradually lost 4.5 kg (10 lb) over the last 4 months; he was unaware that he had been losing weight. History: Ketosis-prone diabetes, Esophageal rupture >1 year ago that required creation of a spit fistula (esophagus to chest wall). (Anastomosis of esophagus to stomach was not possible.). Hospital stay included prolonged immobility. Discharged from hospital to nursing home, and has lived there since. Can now propel himself in a wheelchair and ambulate 100 ft with 1 assistant. Nutrition: Bolus of commercially available solution via gastrostomy tube 4 times daily (unchanged since admission to nursing home). With each tube feeding, he has uncomfortable sensation of fullness and fecal urgency, and passes loose, non-bloody stool. Never skips a feeding, but often ends it before completion because of fecal urgency. He reports no palpitations, night sweats, heat or cold intolerance, anxiety, or depression. Physical examination: unremarkable, beyond weight loss. Lab findings: mild anemia, mild renal insufficiency, hemoglobin A1c is 7.6%. No blood in stool. Which one of the following is the most likely cause of the patient's weight loss? A. Inadequate caloric intake B. Malabsorption C. Occult malignancy D. Inadequate control of diabetes

A. Inadequate caloric intake

A 79-year-old woman has lost approximately 10% of body weight over last 3 months. History: Alzheimer disease (diagnosed 6 years ago), bowel and bladder incontinence. Medications: senna 2 tablets at bedtime. Has lived in nursing facility for 12 months; primarily wheelchair bound but can transfer with assistance; requires assistance with bathing and dressing. Speech is dysarthric and limited to single words. Demeanor is typically pleasant, and she is often observed to be humming or singing. Nutrition: Food intake has declined significantly despite modifications to consistency and content of her diet. Family has provided foods that she previously enjoyed. Staff have set up her tray to facilitate her ability to eat. Which one of the following is the most appropriate next step in managing the patient's weight loss? A. Program of careful hand feeding B. Referral for placement of gastrostomy tube C. Addition of oral nutritional supplements D. Evaluation for occult malignancy

A. Program of careful hand feeding

A 72-year-old black man has right shoulder trauma after he fell in a parking lot. He states that he was walking to his car when he suddenly tripped and fell forward onto his arm. He subsequently learned that he had tripped over a curb. • History: hypertension, high cholesterol, glaucoma Which one of the following most likely contributed to his fall? A. Acute glaucoma B. Failure to use glaucoma drops that morning C. Poor peripheral vision D. Retinal embolus from a carotid plaque

C. Poor peripheral vision

A 65-year-old female comes to the clinic for her wellness visit. Her medical history is significant for hypertension (HTN), hypothyroidism, and tobacco use. Medications include lisinopril and levothyroxine. There are no current concerns and her physical examination is unremarkable. The patient requests a screening test. After review, you determine that the test has a sensitivity of 44% and a specificity of 55% to be performed for a disease with a prevalence of 1.2%. After reviewing the screening test's characteristics, which of the following is the most-appropriate recommendation to give to the patient? Choose the single best answer. A. "I will not order the test as it has a high positive predictive value." B. "I will not order the test as there is a high potential for a false positive result." C. "I will order the test as it has a high positive predictive value." D. "I will order the test as there is a low potential for a false positive result." E. "I will order the test as there is effective treatment available if it is determined the patient has the disease."

B. "I will not order the test as there is a high potential for a false positive result." The ideal screening test should detect a relatively common condition with significant morbidity and mortality, have an adequate screening test sensitivity and specificity, modest consequences of false negative or false positives results, and the availability of an effective treatment for the disease being screened. The test the patient is requesting to order has a low positive predictive value (~4%) and a higher potential for a false positive result. In an asymptomatic patient, screening for a disease that is not prevalent provides no clear benefit to the patient, and may lead to further testing and anxiety for the patient.

An 80-year-old woman describes persistent pain over the top of her right shoulder. The pain began 3 weeks ago. It occasionally awakens her and is worse if she rolls onto her right side. It worsens when she carries more than a few pounds with her right arm. History: type 2 diabetes, coronary artery disease, hyperlipidemia, hypertension, osteoarthritis, stage 3 chronic kidney disease Medications: metformin, hydrochlorothiazide, pravastatin, acetaminophen, tramadol, vitamin D3, calcium carbonate Physical examination: Normal temperature • Blood pressure: 130/70 mmHg, Heart rate: 76 bpm. The patient appears to be in mild distress from shoulder pain. Which one of the following is the most likely cause of the shoulder pain? A. Glenohumeral joint arthritis B. Acromioclavicular joint arthritis C. Supraspinatus tendinosis D. Bicipital tendinosis E. Adhesive capsulitis

B. Acromioclavicular joint arthritis

A 78-year-old man comes to the office because he has pain radiating from his buttock to his right heel. The pain began suddenly 10 days ago. It bothers him in all positions, particularly at night. It limits his walking and his ability to climb stairs. He does not recall similar pain in the past, although he has had intermittent low back pain. Physical examination: Raising the right leg to 40º reproduces the leg pain. Mild weakness of right great toe extensor, right hip abductor, and right hip extensor Full ROM of both hips, with no pain. MRI findings: stenosis at L4-L5 and L5-S1 and moderate disc displacement into right L4-L5 interspace Which one of the following statements is true? A. The pattern of pain is typical for sciatica due to lumbar spinal stenosis. B. Between 50% and 75% of patients report improvement of this pattern of pain within 4 weeks. C. The patient's age is associated with poor prognosis for sciatica. D. The disc herniation indicates a need for surgical intervention.

B. Between 50% and 75% of patients report improvement of this pattern of pain within 4 weeks.

A 78-year-old male is evaluated in a subacute rehabilitation center after sustaining a fall. The fall occurred last night as he was getting out of bed. He does not recall the circumstances of the fall. Medical history is significant for Parkinson disease for which he has taken carbidopa-levodopa for four years. He was hospitalized for community-acquired pneumonia two weeks ago and completed his course of levofloxacin four days ago. During his admission, diphenhydramine was started and he continues to take it at bedtime. Today, his temperature is 37.6C (99.7F), pulse is 85 beats/minute, and blood pressure is 145/80 mmHg. His physical exam reveals lungs clear to auscultation, regular heart rate, no appreciable heart murmur, and mild ecchymosis on his right elbow. He scores 27/30 on a mini-mental state examination (MMSE). He is able to stand and ambulate 20 feet without the use of assistance devices. He is cooperative during the exam. Which of the following most likely triggered this fall? A. Arrhythmia due to levofloxacin B. Delirium due to diphenhydramine C. Dementia secondary to Parkinson disease D. Limited mobility due to Parkinson disease E. Sepsis due to healthcare-associated pneumonia

B. Delirium due to diphenhydramine Medications can be a major risk factor for falls in older patients. For example, postural hypotension can be related to the use of antihypertensive medications. Medications used for other reasons may also cause hypotension or other adverse reactions (e.g. tamsulosin given for benign prostatic hypertrophy). Medications such as diphenhydramine, benzodiazepines, and zolpidem are sometimes prescribed as sleep aids, but these medications may have additional undue effects on older adult patients. Diphenhydramine has been associated with drug-induced delirium, and it is likely that his fall occurred as a result of delirium. His underlying Parkinson disease may be a contributing factor to the fall, but as he is able to stand and ambulate without assistance, it is unlikely that this condition is severe enough to be the main factor. While levofloxacin can cause prolonged QT and increase the risk for cardiac arrhythmias, he is no longer on the medication. His examination is not suggestive of sepsis or dementia.

A 65-year-old male presents to the outpatient office for a routine health evaluation. The patient has a history of hypertension (HTN), for which he takes medications. He has no concerns and his physical examination is unremarkable. During the evaluation, the patient requests an electrocardiogram (ECG) to ensure that his heart is functioning "okay." The United States Preventive Services Task Force (USPSTF) lists obtaining an electrocardiogram in this clinical situation as a Grade D recommendation. Which standard for guideline development does using Grade A - D recommendations follow as per the Health and Medicine Division (formerly the Institute of Medicine [IOM])? Choose the single best answer. A. Articulation of recommendations B. Establishing evidence foundations for and rating strength of recommendations C. Establishing transparency D. Guideline development group composition E. Management of conflict of interest

B. Establishing evidence foundations for and rating strength of recommendations The USPSTF grades its recommendations based on the amount and quality of evidence supporting each intervention. This meets Standard 5, establishing the evidence foundations for and rating the strength of recommendations. Standard 1, establishing transparency, requires that the processes by which a clinical practice guideline is developed and funded should be detailed explicitly and publicly accessible. Standard 3, guideline development group composition, requires that the guideline development group is multidisciplinary, comprises a variety of experts, involves patients and the public, and ensures that strategies are in place to increase the participation of the patient and consumer representatives. Standard 6 (articulation of recommendations), should be articulated in a standardized form, detailing precisely what the recommended action is and under what circumstances it should be performed. In addition, strong recommendations should be worded so that compliance with the recommendation(s) can be evaluated. Standard 2, conflict of interest, ensures that individuals being considered for membership should declare all interests and activities potentially resulting in a conflict of interest with development group activity and divest themselves (and family members) from all financial investments.

A 78-year-old man comes to the office because he is bothered by arthritis in his hands, shoulders, and, to a lesser extent, knees. He is concerned that the osteoarthritis will lead to functional decline. Discussion ensues regarding better pain management, and the potential contribution of osteoarthritis to the development of frailty. What aspect of the frailty phenotype predicts reduced activity? A. Weight loss B. Exhaustion or fatigue C. Decreased strength D. Diminished walking speed

B. Exhaustion or fatigue

A 65-year-old man comes to the office with his wife because she is concerned about his memory. He is a retired engineer. His wife offers examples of recent uncharacteristic mistakes that he has made in their finances. He forgot to pay the mortgage several months ago. In the grocery store, his credit card was denied for missed payments. The patient describes his experience: Balancing accounts feels more effortful and takes longer than it had in the past. He feels overwhelmed by distractions. He is frequently unable to find keys and other objects. He is often unable to recall names of acquaintances until minutes or hours later. Which one of the following is most likely to indicate pathologic neurologic decline? A. Taking longer to complete routine tasks B. Forgetting to pay mortgage and credit card bills C. Having a complaint about memory D. Experiencing difficulty retrieving names

B. Forgetting to pay mortgage and credit card bills

An 80-year-old woman hospitalized after fall and hip fracture • History: stable osteoarthritis of the knees, peripheral neuropathy • Prior to fall, she lived alone and managed all instrumental activities of daily living; occasionally used cane or walker for mild problems with balance • Current status, 4 days after surgery: Needs assistance of 2 persons for transfer; Frequently incontinent of urine; Eating very little Which one of the following is the most likely outcome for this patient 6 months after the fracture? A. Continued slow functional decline B. Recovery to her pre-fracture level of function C. Partial recovery of function D. Stability at the level of function on hospital discharge E. Death

B. Recovery to her pre-fracture level of function

An 82-year-old woman comes in for follow-up related to treatment of depressive disorder. At an appointment 4 months ago, she was tearful and sad. She described loss of appetite, poor sleep, and loss of interest in usual activities. She attributed her symptoms to a change in her living situation: her son had recently moved in with her after he lost his job and got divorced. She had no psychotic symptoms or suicidal ideation. History: depression, diabetes mellitus, hypertension, coronary artery disease, osteoarthritis. No history of suicide attempt or psychiatric hospitalization Medications: metformin, levothyroxine, hydrochlorothiazide, metoprolol, docusate, ibuprofen as needed. Trials of sertraline and venlafaxine were initiated 4 months ago, but she did not tolerate either drug. Mirtazapine was started 3 months ago, with the dosage titrated to 45 mg nightly. She reports better mood and improved sleep and appetite, and she cries less. She still spends most of her time in bed and remains uninterested in usual activities. She is troubled by adverse effects of constipation and weight gain. Laboratory findings: hemoglobin A1c 8%; glomerular filtration rate 35mL/min; normal thyrotropin level Which one of the following treatment options should be considered next? A. Discontinue mirtazapine and initiate tricyclic antidepressant. B. Refer for cognitive-behavioral therapy. C. Add aripiprazole. D. Add lithium. E. Refer for electroconvulsive therapy.

B. Refer for cognitive-behavioral therapy.

An 85-year-old man with 2 episodes of low-volume urinary incontinence in the last year, when he "waited too long" • Functional assessment: Near-visual acuity of 20/50 on the right and 20/60 on the left with correction; Fails whisper test in right ear ; Gait speed at usual pace is 0.9 m/sec ; Mini-Cog screen: 2 of 3 items on recall, completes an accurate clock 36 CASE 3 (2 of 3) Which one of the following is the most appropriate next step for evaluation of functional disability? A. Refer to audiology. B. Refer to ophthalmology. C. Refer to physical therapy. D. Complete additional cognitive testing. E. Complete an incontinence evaluation.

B. Refer to ophthalmology.

A 73-year-old woman comes to the office for followup. She was last seen 1 week ago, a few days after her husband's death from end-stage renal disease and metastatic cancer. She had been his sole caregiver for the last 3 years and handled all medical and practical decisions in his last days. She was distressed and barely able to speak. She was sleeping poorly, had lost weight, and felt isolated and unable to function. She declined medication to help her sleep and agreed to return in a few days for follow-up. Today, she is calmer, but she still cries easily, cannot sleep, and is unable to concentrate. She realizes that she neglected her health during her husband's illness and wants to start taking care of herself, but she has a feeling of futility at the idea of engaging in any activity. She states that she has no active or passive suicidal ideation. She has no children or other family, and she neglected friendships because of the demands of caregiving. History: hypertension, osteoarthritis, hypercholesterolemia Which one of the following is the most appropriate course of action? A. No treatment is necessary for normal grief reaction. B. Start treatment with an antidepressant if she shows no improvement in 2 weeks. C. Consider psychiatric diagnosis unrelated to grief. D. Encourage her to start seeing friends.

B. Start treatment with an antidepressant if she shows no improvement in 2 weeks.

An 82-year-old woman comes in for follow-up related to treatment of depression. She has been taking mirtazapine because she did not respond to sertraline and venlafaxine. She has had a partial response to mirtazapine, despite adequate dosage and duration of therapy. She continues to spend much time in bed, is deconditioned, and has fallen twice. Since beginning therapy, she has gained weight and is finding it difficult to control her diabetes. During today's visit, she describes hopelessness and sometimes believes that she would be better off dead. She denies any intent or plan to kill herself. She reports frequent arguments with her unemployed son about finances. There is no skilled provider for cognitive-behavioral therapy in the area. Which one of the following treatment options should be considered next? A. Hospitalization and electroconvulsive therapy B. Referral for transcranial magnetic stimulation C. Augmentation with bupropion D. Switch from mirtazapine to duloxetine E. Referral for family therapy

C. Augmentation with bupropion

An 88-year-old woman comes to the office for an urgent visit because, when she awoke this morning, she found that she could no longer see to read. She depends on her left eye for vision because she lost her right eye in an accident when she was a child. • On examination, vision is 20/400 in the left eye. She does not perceive light in the right eye. She is referred emergently to an ophthalmologist, who diagnoses neovascular age-related macular degeneration. Which one of the following is the most appropriate therapy? A. Topical prostaglandin analog B. Phacoemulsification with intraocular lens implantation C. Intravitreal vascular endothelial growth factor inhibitor D. Glasses with updated refraction E. Oral lutein, zeaxanthin, and omega-3 eye nutrients

C. Intravitreal vascular endothelial growth factor inhibitor

An 80-year old male with coronary artery disease, diabetes mellitus type 2, and hypertension presents after a fall he had at home about five hours ago. He felt well this morning. After a light breakfast, he read the newspaper at the kitchen table. When he stood up, he felt lightheaded and fell to the floor. His daughter, who lives with him, heard the thud and found him on the floor. He was orientated to person, place, and time when found by his daughter. He denied having any urinary or stool incontinence. He also has not had fevers, palpitations, abdominal discomfort, hematuria, or dysuria. As he was complaining of left hip pain, she brought him into the emergency room for evaluation. He has been prescribed aspirin, metoprolol, enalapril, amlodipine, and metformin, all of which he took this morning. The enalapril was recently added to his regimen. On exam, vitals were as follows: temperature is 37.6 C (99.7 F), pulse is 60 beats/minute, and blood pressure is 110/70 mmHg. You find a pleasant male who appears his stated age and is in no distress. His exam, including the neurological exam, is unremarkable except for ecchymoses over the left hip. He scores 29/30 on the mini-mental state examination (MMSE). He is unable to complete a "Get Up and Go" test due to pain in his left hip. Initial tests show a finger stick glucose of 220 mg/dL, sodium 135 mmol/L, creatinine 1.0 mg/dL, white blood cell count 9,000 cells/mL, hemoglobin 14 g/dL, and troponin I 0.01 ng/mL. A urinalysis showed 4+ bacteria with no leukocyte esterase or nitrites. An electrocardiogram (ECG) showed a normal sinus rhythm with significant Q waves in leads II, III, and avF. A non-contrast computed tomography (CT) of the head shows no structural lesions. A pelvic radiograph and magnetic resonance imaging (MRI) of the pelvis both show no evidence of fracture. What is the most likely contributor to his fall? A. Acute coronary syndrome B. Hypoglycemia C. Medications D. Seizure disorder E. Urinary tract infection

C. Medications Medications are an important risk factor contributing to falls in older people. In this case, orthostatic hypotension related to the recent initiation of enalapril was likely causative. Adverse reactions due to specific types of medications, greater number of medications (polypharmacy), and recent changes in medication dose have been associated with an increased risk of falls. Medication use is one of the most-modifiable risk factors for falls. Other risk factors for falls include visual impairment; cognitive impairment; a history of falls; chronic diseases such as osteoarthritis, diabetes mellitus, and cardiovascular disease; and environmental factors such as loose rugs and lack of handrails. Hypoglycemia is unlikely to have contributed to this fall. His only medication for diabetes is metformin, which does not cause hypoglycemia; in addition, he would not have normal mentation immediately after the fall if it was secondary to hypoglycemia. Likewise, the lack of a postictal state makes a seizure disorder unlikely. He is lacking signs and symptoms of a urinary tract infection, suggesting an asymptomatic bacteriuria that does not require intervention and would not result in a fall. While Q waves are seen on the ECG, these reflect an old myocardial infarction. The normal troponin I result also speaks against a myocardial infarction as an etiology of his fall.

An 80-year-old female presents to the office with urinary incontinence. She has a history of congestive heart failure (CHF) and severe osteoarthritis of her knees. She takes furosemide, but states that she often misses doses because it makes her urinate more frequently. She is embarrassed when she has urinary incontinence while out of the house. She has no dysuria or difficulty controlling her bowel movements. She can feel the urge to urinate, and had rare difficulty with incontinence prior to starting diuretics. Her exam is significant for clear lungs, severe crepitus of both knees, and a normal neurologic exam. Her "Timed Up and Go" test is 30 seconds. What is the next step in the management of her urinary incontinence? A. Discontinue diuretics B. Initiate tamsulosin C. Modify the timing of her diuretic therapy D. Order a computed tomography (CT) of her abdomen and pelvis E. Place an indwelling urinary catheter

C. Modify the timing of her diuretic therapy This patient has functional incontinence. She has bladder control, but because of her severe osteoarthritis, she is unable to get to the bathroom on time. This is an indication for modifications to her medications or physical environment. Discontinuing diuretics may decrease the frequency of urination, but is likely to trigger a CHF exacerbation. Therefore, changing the timing of her diuretics is likely the safest option. Consider recommending that she take her diuretic first thing in the morning, plan to have easy access to a bathroom for the following six hours, and then schedule social engagements later in the day once the potent diuretic effect has passed. Tamsulosin is helpful in males with benign prostatic hyperplasia (BPH) and overflow incontinence, but not in women with functional incontinence. A CT scan would provide unnecessary radiation, and not manage the underlying problem. An indwelling catheter increases the risk of a urinary tract infection (UTI), delirium, and falls in older adults and is not indicated.

A 77-year-old female is brought to the office by her daughter with complaints of fatigue, poor appetite, and forgetfulness that have been getting progressively worse over the past three months. Conversing with the patient is difficult, as she only provides one-word answers to your questions; her daughter provides most of the history. The patient has had weight loss and constipation for which she has been taking glycerin suppositories. She has had more difficulty bathing and decreased motivation to practice good hygiene. She lives alone, but her daughter is nearby and has been visiting her with increasing frequency over the past month. On physical examination, she is afebrile. Her pulse is 120 beats/minute. Her blood pressure is 140/90 mmHg. She appears frail and disheveled. A mini-mental state examination (MMSE) is performed, and her score is 7. What would be the ​next best step in management? A. No further lab testing needed. Start donepezil. B. Order a serum folate level C. Order a thyroid stimulating hormone (TSH) level D. Order a transthoracic echocardiogram E. Order an erythrocyte sedimentation rate (ESR)

C. Order a thyroid stimulating hormone (TSH) level Goal: Identify atypical manifestations of hyperthyroidism as a reversible cause of dementia in older adults. In an older adult patient with a new diagnosis of dementia, a thorough evaluation of reversible causes of dementia should be conducted. The American Academy of Neurology (AAN) recommends that patients be screened for depression and be tested for possible underlying thyroid disease and vitamin B12 deficiency. While hypothyroidism is more common as a cause for dementia, as it can lead to cognitive slowing, hyperthyroidism is also associated with dementia. Graves disease is the most likely underlying cause of​ hyperthyroidism. Hyperthyroidism may present atypically in older adult patients. Referred to as "apathetic hyperthyroidism," it can present without the common symptoms of hyperactivity, tremor, or other manifestations of sympathetic overdrive. A goiter is less likely in older patients. Tachycardia may be present in about 60 percent of older hyperthyroid patients. Folate deficiency is not associated with dementia syndromes. While she may ultimately require an echocardiogram to diagnose the etiology of her tachycardia, an initial evaluation should involve an electrocardiogram to define her underlying rhythm. ESR is a non-specific marker of inflammation and will add little to the evaluation of secondary causes of dementia.

A 79-year-old woman comes to the office because she has pain predominantly in the right buttock. The pain is worse when she stands or walks. She has a pronounced limp, because she is unable to bear weight on her right leg without significant pain. MRI of the lumbar spine 6 months ago showed diffuse degenerative changes throughout the lumbar area, with significant stenosis at L4-L5 and L5-S1. The patient has been treated by a pain specialist with a series of facet joint injections, without significant relief. Which one of the following is the most appropriate next step? A. Bone scan to exclude tumor of the spine B. Referral for decompression laminectomy of the lumbar spine C. Passive examination of the right hip D. Trial of tramadol 25 mg titrated to 4 times daily as needed E. Referral for intensive program of physical therapy

C. Passive examination of the right hip

An 85-year-old woman comes to the office for routine evaluation. Found to be occasionally incontinent of urine. Sometimes gets dizzy when she stands. Fell 6 months ago, without injury. History: hypertension, declining memory (over past 3 years) Medications: hydrochlorothiazide, donepezil, quetiapine Physical examination: Heart rate, 60 bpm; blood pressure, 150/80 mmHg seated and 130/70 mmHg standing. She is thin and mildly cachectic. Which one of the following is the single most predictive risk factor for this patient's future fall risk? A. Dementia B. Hydrochlorothiazide C. Prior fall D. Orthostatic hypotension E. Quetiapine

C. Prior fall

An 80-year-old woman fell and underwent repair of hip fracture 3 days ago. She now reports dizziness, and the nurse notes excessive daytime drowsiness. History: hypertension, frequent falls, post-herpetic neuralgia. Pre-admission medications have been restarted. Hydrochlorothiazide 12.5 mg/d, extended-release metoprolol 50 mg/d, amlodipine 10 mg/d, gabapentin 600 mg three times daily, calcium carbonate 500 mg three times daily. Medications started after surgery: Subcutaneous enoxaparin 30 mg/d, daily multivitamin, docusate 250 mg twice daily, senna 8.6 mg twice daily Oxycodone 5-15 mg every 4 hours as needed for pain; she received two 10-mg doses in the last 24 hours. Examination • Weight 45 kg (99 lb), blood pressure 144/76 mmHg, with no orthostatic changes • Estimated creatinine clearance (based on laboratory samples drawn today) is 30 mL/min. •Two months ago, estimated clearance was 60 mL/min. Which one of the following is the best next step in management? A. Discontinue oxycodone. B. Increase enoxaparin to 30 mg twice daily. C. Reduce gabapentin to 600 mg twice daily. D. Start alendronate 70 mg once weekly.

C. Reduce gabapentin to 600 mg twice daily.

A 79-year-old man requests a second opinion on management of prostate cancer. His urologist recommended prostate biopsy because his PSA level is 6.4 ng/mL. Pathology report describes low-volume adenocarcinoma of the prostate. Gleason score is 6 (3+3). The patient is generally healthy. Both parents died in their late 80s. History: Hypertension, hypercholesterolemia. Mild lower urinary tract symptoms, managed with an α-blocker. Digital rectal examination: markedly enlarged, otherwise benign prostate gland Which one of the following is the most appropriate recommendation? A. External beam radiotherapy B. Transurethral radical prostatectomy C. Routine follow-up with PSA tests D. Androgen deprivation therapy

C. Routine follow-up with PSA tests

An 83-year-old man comes for follow-up after catheterization in the ED for acute urinary retention. Catheterization yielded 450 mL urine. At follow-up, he describes recent increasing difficulty with urination and worsening constipation. He had previously reported mild lower urinary tract symptoms that were not bothersome. He recently began taking an OTC "multisymptom relief" product for a cold. He is independent in ADLs and IADLs. Which one of the following should be included in the immediate treatment plan? A. Start an α-blocker. B. Start a 5α-reductase inhibitor. C. Stop the over-the-counter medication. D. Refer for urodynamic studies.

C. Stop the over-the-counter medication.

An 85-year-old woman has fallen twice in the last 6 months. Most recent fall resulted in a hip contusion, from which she has now recovered. A personal trainer works with her 3 days per week doing gait and balance training. History: diabetes (controlled by diet), osteoarthritis Surgical history: cholecystectomy, removal of a cataract in her right eye Medication: acetaminophen as needed Physical examination: Blood pressure, 148/80 mmHg (seated and standing). Mild bilateral hearing loss. Visual acuity: 20/30 on the right and 20/80 with correction on the left Based on the results of controlled clinical trials, which one of the following interventions is most likely to help prevent future falls in this patient? A. Removing the second cataract B. Wearing hip protectors C. Taking a vitamin D supplement D. Wearing non-slide shoes E. Using a personal alarm system

C. Taking a vitamin D supplement

A 75-year-old woman comes to the office because she has pain during intercourse. She was widowed 28 years ago and had no sex partners until her recent remarriage. She had no difficulty with intercourse during her first marriage. She has no other vaginal complaints. History: Hysterectomy and bilateral oophorectomy at age 52 for uterine cancer. Postoperative treatment was not necessary. On the advice of her gynecologist, she did not take estrogen therapy. Her 2 children were delivered by cesarean section. Physical examination: No vulvar lesions. The vaginal mucosa is pale with visible veins. The introitus is mildly stenotic. Vaginal caliber and length are reduced. Which one of the following is the most appropriate management? A. Vaginal lubricant (eg, K-Y® jelly) B. Referral for possible vaginoplasty C. Topical vaginal estrogen D. Vaginal dilator E. Ospemifene

C. Topical vaginal estrogen

A 75-year-old man started α-blocker therapy for lower urinary tract symptoms. He is tolerating therapy well, with no adverse effects. Urinary hesitancy and need to strain have largely resolved. Force of stream is improved. He remains bothered by frequency with small volumes, particularly at night, and would like further treatment. There are no confounding medical problems, and he appears healthy. Examination: Normal genitalia. Moderately enlarged, benign prostate. Postvoid residual (obtained in office): 75 mL Which one of the following is the best approach for managing his persistent symptoms? A. Add finasteride. B. Order intermittent catheterization to empty his bladder completely. C. Refer to urology for consideration for transurethral resection of the prostate. D. Add an antimuscarinic agent.

D. Add an antimuscarinic agent.

An 80-year-old woman comes to the office with her daughter. The daughter is concerned that her mother has recently begun to withdraw from former activities (attending book clubs, lectures). • Patient's self-reported health status has changed from "excellent" last year to "average" today. • During the appointment she misunderstands some questions, and she appears to struggle to understand some of what is said. Her memory does not seem as keen as on previous visits. • In response to direct questions, she states that over the past 2 weeks she has felt a little down. • She says that if people would include her in conversation, she would probably feel better and be more inclined to engage in the activities she previously enjoyed. • Medications: daily multivitamin, daily aspirin • Physical examination: Normal blood pressure and EKG; MMSE score is 26 of 30; Findings from routine laboratory tests are normal. Which one of the following is the most appropriate next step? A. Begin antidepressant therapy. B. Refer for psychiatry consultation. C. Refer for neurologic evaluation. D. Administer in-office hearing screen.

D. Administer in-office hearing screen.

A 30-year-old female comes to the clinic for a wellness exam. The patient is previously healthy with a family medical history of obesity, hypertension (HTN), hyperlipidemia, and diabetes mellitus. She is on no medications. She endorses decreased physical activity and increased cigarette smoking over the past five months. Vital signs are normal. Physical examination is normal, including normal blood pressure. When asked if she is interested in quitting smoking, she suggests she might be ready, but does not know if she has the willpower. However, she acknowledges that it is harmful to her health. What can be done to help her quit? Choose the single best answer. A. Agree that smoking is harmful to her health and advise that she set a quit date. Advise follow-up. B. Arrange a follow-up date to discuss strategies for successfully quitting smoking. Advise her to consider in the interim. C. Ask if she has friends who have quit in the past and advise she contact friends for solutions that have worked in the past. Ask her to follow up once she has quit. D. Assess if she knows what situations increase her tendency to smoke and assist on solutions to avoid these scenarios in the short term. Agree on a quit date together. Arrange follow-up. E. Assist her in understanding why she is smoking and advise her to stop. Ask her to set a quit date and notify you soon.

D. Assess if she knows what situations increase her tendency to smoke and assist on solutions to avoid these scenarios in the short term. Agree on a quit date together. Arrange follow-up. Though several of these may be reasonable in a given scenario, it is most appropriate to assess her own barriers to success and assist the patient on solutions towards overcoming these barriers, while agreeing together on a plan for a quit date and a follow-up plan at that time. All other answer choices do not help facilitate overcoming of barriers in place and do not help facilitate readiness to change.

A 68-year-old woman describes vision problems. • Progressive blurring of distance vision • Increased difficulty with nighttime driving, which she attributes to worsening glare and haloes around lights • She no longer needs glasses for reading. • History: hypertension, hypercholesterolemia Which one of the following is the most likely cause of the changes in her vision? A. Uncontrolled hypertension B. Poorly controlled blood glucose C. Choroidal neovascularization D. Cataract progression E. Increased intraocular pressure

D. Cataract progression

A 73-year-old man has memory problems that are increasingly evident to his wife, and he has had several unexplained falls. Neuropsychiatric history: Seven years ago he began to have impaired smell, altered taste, and fitful sleep with recurrent dream enactment. Symptoms have progressed. The dream enactments wake his wife at night. Fluctuating cognitive dysfunction and bilateral arm tremors developed. For several years, he has had hallucinations of children in the room. The hallucinations do not frighten him. The hallucinations worsened when he was hospitalized for a UTI last year. Neuroleptic agents were administered, but his condition deteriorated and he required restraints for several hours. Behavioral interventions have been unsuccessful. History: hypertension, hyperlipidemia, diabetes, urinary incontinence, constipation. MRI of the brain shows mild white matter changes. Which one of the following medications should NOT be considered for this patient? A. Clonazepam B. Donepezil C. Fludrocortisone D. Haloperidol E. Rivastigmine

D. Haloperidol

A 75-year-old man has age-related hearing loss, diagnosed by an audiologist. The patient requests advice regarding hearing interventions. • He has difficulty understanding speech at meetings and in restaurants; he describes "hearing people talk but not making out the words." • Score on the Hearing Handicap Inventory for the Elderly (short form) is 25, consistent with significant self-reported handicap. (HHIE-S is a self-reported questionnaire that quantifies the handicapping effects of hearing loss.) • Pure-tone test indicates a mild sloping to moderate sensorineural hearing loss most pronounced in high frequencies. • Speech understanding in quiet environment is excellent, but decreases to fair in a noisy environment. Which one of the following is the most suitable intervention for this patient? A. Cochlear implant B. Bone-anchored hearing aid C. Environmental modifications D. Hearing aid and counseling on communication strategies

D. Hearing aid and counseling on communication strategies

An 81-year-old woman is admitted to a nursing home after 6 weeks in hospital for pneumonia and acute respiratory failure. She is deconditioned and needs assistance with all ADLs. Hospital course: Intubation and ventilatory support for 3 days. Complications: acute kidney injury requiring dialysis, prolonged ileus, pressure ulcers, UTI, C. difficile-induced colitis, mild right-hemisphere stroke. At discharge, all catheters, IV lines, and antibiotics were discontinued. She can ambulate short distances. She tolerates a regular diet. No difficulty chewing or swallowing, per recent assessments by dietitian and speech therapist Physical examination (at nursing home) Vital signs, chest, cardiac, and abdominal findings: unremarkable. Weight 82 kg (181 lb) (pre-hospitalization: 81.5 kg). Pretibial and presacral edema 3-4+. Unstageable 5 cm × 4 cm sacral pressure ulcer surrounded by non-inflamed, healthy skin. Muscle strength: mild weakness of left upper extremity. Laboratory findings • Hemoglobin 10.5 g/dL • WBC 7.8/µL • Creatinine 1.8 mg/dL • BUN 12 mg/dL • Albumin 2.7 g/dL Which one of the following is most appropriate for optimizing the patient's nutritional status? A. Start megestrol acetate, 400 mg orally twice each day. B. Order a complete swallowing assessment. C. Place a feeding tube and start enteral feedings with a high-protein polymeric (nutritionally complete) formula at a rate sufficient to provide 30 kcal/kg daily. D. Provide feeding assistance and monitor nutrient intake with complete calorie counts for the next 3 days. E. Start a high-protein oral nutritional supplement, 240 mL three times daily between meals.

D. Provide feeding assistance and monitor nutrient intake with complete calorie counts for the next 3 days.

A 23-year-old male presents to the clinic for his wellness visit. He has no concerns. His family history is negative. He is not taking any medications. He appears well when you enter the room. His vital signs are normal, including a blood pressure of 120/70 mmHg and a body mass index (BMI) of 24 kg/m2. His physical examination is normal. His last three wellness exams have included labs with normal results for his complete blood count (CBC), lipid panel, and comprehensive metabolic profile (CMP). He received his last vaccines at 11 years old when he was given tetanus and meningococcal immunizations. Which of the following is the best preventive care measure for this patient? Choose the single best answer. A. Order a lipid panel B. Perform an electrocardiogram (ECG) C. Prescribe a multivitamin D. Provide tetanus, diphtheria, and pertussis (Tdap) and human papillomavirus (HPV) vaccines E. Refer to a nutritionist

D. Provide tetanus, diphtheria, and pertussis (Tdap) and human papillomavirus (HPV) vaccines Administration of immunizations such as the Tdap and HPV vaccines helps to prevent disease, which is one of the major categories of high value preventive care. Yearly monitoring of a lipid panel is not recommended for individuals with low risk for abnormalities. The American College of Cardiology/American Heart Association (ACC/AHA) guidelines recommend measuring traditional risk factors, including a lipid panel, every four to six years. An ECG is not routinely recommended in people who have no risk factors or symptoms of a cardiovascular problem. Multivitamins are not routinely indicated for someone who does not have risk factors for nutrition deficiencies. A nutritionist referral is not indicated for someone of normal health without a nutritional deficiency, cardiovascular problem, or an abnormal BMI.

An 81-year-old man vomited overnight and has since had a nonproductive cough, rapid breathing, and fever. History: severe intellectual disability and schizophrenia; gastrostomy tube because of severe dysphagia. Lives in a nursing home and is dependent for all activities of daily living. He does not speak. Physical examination: Appears thin and chronically ill; no sign of respiratory distress. Temperature: 38.5°C (101.3°F) • Heart rate: 85 bpm • Blood pressure: 115/70 mmHg • Respiratory rate: 24 bpm • O2 saturation 91% on room air. Cardiac auscultation: regular rhythm and rate, no murmur. Lung sounds are coarse throughout. Laboratory findings: WBC 19.2/µL, with 77% neutrophils and 9% bands Hemoglobin10.1 g/dL Hematocrit 31% Platelets 258,000/µL Sodium 139 mEq/L Potassium 3.9 mEq/L Chloride 100 mEq/L Carbon dioxide 26 mEq/L BUN 29 mg/dL Creatinine 0.44 mg/dL. Chest radiography shows diffuse mild interstitial prominence, unchanged from prior examinations. Which one of the following is the most appropriate first-line treatment? A. Ceftriaxone 1 g IM and azithromycin 500 mg via gastrostomy tube B. Clindamycin 450 mg three times daily via gastrostomy tube C. Prednisone 60 mg via gastrostomy tube D. Supplemental oxygen

D. Supplemental oxygen

A 68-year-old woman wants to discontinue hormone replacement therapy. She is concerned because her mother has had a stroke and a friend has breast cancer. She has taken estrogen plus progestin since age 50, because she was miserable during perimenopause. She has tapered her dosage to every other day, but now she is bothered by frequent hot flushes that occur day and night. Which one of the following would be most effective in helping her taper off hormone therapy? A. Paroxetine 7.5 mg/d orally B. Pregabalin L 75 mg/d orally C. Extended-release venlafaxine 75 mg/d orally D. Clonidine 0.5 mg/d orally E. Any of the above

E. Any of the above

An 81-year-old woman had surgery for a ruptured appendix 6 days ago. She responded well to IV fluids and antibiotics and has been afebrile for 4 days. She is now on a regular diet. Per nurses' notes, she consumes 50%-75% of her meals. Past medical history: unremarkable Physical examination Alert, oriented, in no acute distress. Weight: 79.5 kg (175 lb) (5 kg (11 lb) above weight on admission). Body mass index: 28 kg/m2. Lungs are clear. Jugular venous pressure: <7 cm. 3-4+ pretibial and presacral edema (not present at admission). Surgical wound in right lower quadrant of abdomen. Laboratory findings: normal for electrolytes, BUN, creatinine, WBC Which one of the following would be most helpful for assessing the adequacy of the patient's nutrient intake? A. Serum albumin level B. Serum prealbumin level C. Daily weighing D. Resting metabolic rate measured by indirect calorimetry E. Complete calorie counts for the next 3 days

E. Complete calorie counts for the next 3 days

A 77-year-old man is evaluated because he fell in the nursing home. He did not sustain any significant injuries. History: hypertension, atrial fibrillation, insomnia Physical examination: blood pressure, 140/80 mmHg seated and 120/70 mmHg standing; heart rate, 68 bpm and irregularly irregular Medications: amlodipine, vitamin D, warfarin, amiodarone, lorazepam Which one of his medications is most strongly associated with falls? A. Amlodipine B. Vitamin D C. Warfarin D. Amiodarone E. Lorazepam

E. Lorazepam

Which one of the following micronutrient supplements increases healing of pressure ulcers in older adults? A. Zinc sulfate B. Selenium C. Vitamin C D. Daily multivitamin that contains zinc, selenium, iron, and vitamins A, C, and E E. No supplement is necessary if vitamin deficiency is not present.

E. No supplement is necessary if vitamin deficiency is not present.

An 85-year-old woman comes to the office to establish care. History: hysterectomy in Ecuador for cancer at age 65. She thinks it was cancer of the uterus but is uncertain. Her presenting symptom was vaginal bleeding. She did not receive chemotherapy or radiation therapy. •She had regular follow-up until age 75, but none since then because of financial hardship. (She is ineligible for Medicare.). Her family can now pay for her health care. They request tests to exclude cancer recurrence. Which one of the following is the most appropriate posttreatment surveillance for this patient? A. Pap smear B. Ultrasonography of the pelvis C. Measurement of CA-125 D. Chest radiography E. Pelvic examination

E. Pelvic examination

An 84-year-old woman, lives independently, concerned about driving. On 2 occasions, she had minor collisions when she was backing up her car (she dislodged rearview mirror and struck a fire hydrant, each time on the passenger side). • History: osteoarthritis of the cervical and lumbar spine and mild hearing impairment • Medications: acetaminophen and calcium plus vitamin D • Functional assessment: intact cognition and vision, with near-visual acuity 20/30 in each eye. She fails the whisper test. • Physical examination: Decreased range of motion in neck and back in all planes (unchanged from previous findings); Diffuse osteoarthritic deformities in the knees and hands; Decreased range of motion at the hips; Mild kyphosis; Motor strength and deep tendon reflexes are symmetric and intact Which one of the following is the most appropriate next step? A. Obtain radiography of the cervical spine in 3 views. B. Recommend that she no longer drive. C. Refer to physical therapy. D. Refer to audiology. E. Refer to occupational therapy.

E. Refer to occupational therapy.

A 52-year-old female comes to the clinic for her wellness visit. Medical history is significant for gestational diabetes with both of her children, but is otherwise healthy. Her family history is only significant for hypertension (HTN). Her physical exam is normal. She has been resistant to screening mammograms because she feels her risk is low as she has no family history of breast cancer. Which of the following should be considered when applying breast cancer screening guidelines to counsel this woman? Choose the single best answer. A. The guidelines are based on one article with moderate quality evidence. B. The guidelines are internally reviewed by a well-known radiology center. C. The guidelines are updated every 20 years. D. The guidelines are utilized in a reputable hospital. E. The guidelines have a method to rate their strength of recommendations.

E. The guidelines have a method to rate their strength of recommendations. This patient is 53 years old and therefore falls in the age group where routine mammography is recommended even without a family history of breast cancer. When counseling your patients it's important to determine if the preventative guidelines are reputable. There are 8 standards that should be considered during guideline development, including: Establishing transparency Management of conflict of interest Guideline development of conflict of interest Clinical practice guidelines—systematic review intersection Establishing evidence foundations for and rating strength of recommendations Articulation of recommendations External review Updating


Conjuntos de estudio relacionados

Chapter 7.1 ¿Que han hecho? (Present Perfect)

View Set

Psychiatric-Mental Health Practice Exam HESI

View Set

LearningCurve 6a. Basic Concepts of Sensation and Perception

View Set